Managing heat dissipation in a magic wandExplaining where energy comes from to power magicMagically knock someone down and pin them using ScienceMagic vs. modern technologyLimitations of magic as an energy conversion processWhat could hard light be?Could stone be used in place of metal for certain items?Magic System Balance QuestionPhysical consequences and safety concerns with magic light spells?What are the consequences of wishing for 'all' the gold?Gravity magic - How does it work?

Divisor Rich and Poor Numbers

How to pipe results multiple results into a command?

Was murdering a slave illegal in American slavery, and if so, what punishments were given for it?

How to draw pentagram-like shape in Latex?

How was the blinking terminal cursor invented?

What color to choose as "danger" if the main color of my app is red

Hotel booking: Why is Agoda much cheaper than booking.com?

how to create an executable file for an AppleScript?

Are there any symmetric cryptosystems based on computational complexity assumptions?

Have GoT's showrunners reacted to the poor reception of the final season?

How to laser-level close to a surface

How does this piece of code determine array size without using sizeof( )?

Alternative classical explanation of the Stern-Gerlach Experiment?

Does the US Supreme Court vote using secret ballots?

How to say "that" as in "the cow that ate" in Japanese?

Largest memory peripheral for Sinclair ZX81?

Windows reverting changes made by Linux to FAT32 partion

Save my secrets!

Why is Drogon so much better in battle than Rhaegal and Viserion?

Why use a retrograde orbit?

How can sister protect herself from impulse purchases with a credit card?

Why would you put your input amplifier in front of your filtering for an ECG signal?

Is there any deeper thematic meaning to the white horse that Arya finds in The Bells (S08E05)?

Why would company (decision makers) wait for someone to retire, rather than lay them off, when their role is no longer needed?



Managing heat dissipation in a magic wand


Explaining where energy comes from to power magicMagically knock someone down and pin them using ScienceMagic vs. modern technologyLimitations of magic as an energy conversion processWhat could hard light be?Could stone be used in place of metal for certain items?Magic System Balance QuestionPhysical consequences and safety concerns with magic light spells?What are the consequences of wishing for 'all' the gold?Gravity magic - How does it work?













9












$begingroup$


Backstory



You won't believe it, but I'll tell you anyway. I'm a Level 61 wizard. If you think about it, wands aren't all that hard to use. Point, invoke, joyfully watch your enemy burn in the fires of Reetath. I've watched wizards do it all my life.2



So, when I stumbled across a fairly new-looking wand,3 I picked it up. Wands are fairly generic, right? They all shoot little darts, lightning bolts, fireballs... We've all heard the stories.4 So I wanted to see what I could do! I visited the local library and asked the librarian5 what the word was for "fireball." Then skipped6 to an open field (I wasn't looking to roast a cow or anything), pointed my new wand at some unsuspecting tree, and said Lamboobalar!



The next thing I remember was waking up in my mom's house7 with my hand wrapped up to my elbow and wishing someone would hit me behind the ear with a hammer. It hurt! And what I want to know is, what'd I do wrong?



World Rules



  • Wands are a bit like magical rifles. Fire a bullet through a rifle and the barrel heats up. Fire enough bullets fast enough and the barrel becomes burning hot! No matter what you cast with your wand, the more powerful the spell or the faster you cast the hotter the little honker will get.

  • Magic is the funniest thing. The wand must be held. Not touched by skin necessarily, but held. If you set a wand on a table and scream Wapatay! the wand will happily sit there and ignore you. If you pick up the table, it'll still ignore you. But! grasp it with some chopsticks or with a gauntlet and BOOM! pink mist!8

  • As a bit of a reference point, casting the fireball spell generates enough heat to cause 3rd degree burns on the unprotected hand and wrist. Using the wand to magically rap someone's knuckles, causing them to drop their sword, would only cause noticeable heat if you were paying attention. Casting the dread Sheeeaaaaah-Moogatee-Hah! spell, which consumes your enemy's castle in a somewhat makeshift volcano, will cause an intensely bright light followed immediately by the unprotected wielder converting into a lovely cloud of carbon.9

  • For the purpose of this question, the wand is indestructible. You can't drill through it or cut slots into it, either. The surface has friction similar to any smooth hardwood and does taper from the base to the tip. It's about 1.5cm (5/8") at the base and 0.6cm (1/4") at the tip.

Question Limitations



  • You cannot use magic to solve the problem. Using magic to solve a problem that's a consequence of magic would be like trying to put out a fire with a bucket of gasoline.10 That's why the science-based tag has been applied. No magic!


  • If you're thinking, "this is just a heat-sink problem!" you're exactly right! Good luck!


  • The wand must be usable in a melee situation. In other words, you can't simply encase the wand in a 2-foot diameter column of iron sitting in a custom-designed cart wherein the wizard sits comfortably on a velvet stool while embracing the rod bar. (a) while that would probably work per my rules, it's not practical in a melee situation and (b) the wand would probably blow a hole in the front of the bar, spewing sub-vapor-point iron all over the place. It would look spectacular — and probably fry the wizard anyway. So, the wizard must carry the wand and can't be more encumbered than a suit of plate mail.


  • You may only use 13th century technology (my sincere apologies to TimBII for not stating this here, I can see where it was easy to overlook).



Question: Using 13th century technology, what can I do to protect my erstwhile wizard when he casts a fireball spell with this wand?



Bragging rights to answers that point out how to avoid the cloud of carbon problem — but that's optional.




1Fine! I'm a level 1 wizard... OKAY! I'm some dumb schmuck who found the wand in a gutter! So sue me.



2YES! There are wizards in Grubda! Well there only had to be one! It didn't LOOK that hard! If it means that much to you, ask your own @#*& question! Sheesh, everybody's a critic!



3That body on the sidewalk had nothing to do with the wand. Yes... I'll swear on my mother's grave! Now let me finish!



4Except you! Now shut up!



5You know she's my sister... right?



6Yes, I skipped! Like a 6-year-old girl! I was excited, alright? What would YOU do if you found a wand? Turn it in to the constabulary!? Are you nuts!?



7Don't say it! You were thinking it! Yes you were! I'm saving up for my first horse, alright?



8And molten metal thinly plating everything in a 3-meter radius. Wapatay! is not for the faint of heart.



9Magic should always have a price, don't you think?



10This is obvious, right? Just because it's liquid doesn't mean it'll act like water? Yeah.










share|improve this question











$endgroup$







  • 1




    $begingroup$
    At the time of this comment, no-one seems to have applied a detailed eye to the necessary solution. I'll offer a bounty on this question if it still doesn't get a proper answer after 24 hours. 10 Yep.
    $endgroup$
    – Hoyle's ghost
    6 hours ago







  • 1




    $begingroup$
    Is this supposed to be a question or a puzzle? o_O
    $endgroup$
    – Jesse Amano
    5 hours ago










  • $begingroup$
    @JesseAmano Yeah, I’m pretty sure that this belongs on the Puzzling StackExchange site, given that the OP seems to be playing a game of “read my mind and find the exact solution I’m thinking of” with the answerers.
    $endgroup$
    – nick012000
    4 hours ago










  • $begingroup$
    @JesseAmano Puzzle? How odd? This is a question asking for the application of 13th century technology given the rules of my world. No, it is not a puzzle.
    $endgroup$
    – JBH
    2 hours ago






  • 1




    $begingroup$
    Does this world have dragons?
    $endgroup$
    – Xavon_Wrentaile
    1 hour ago















9












$begingroup$


Backstory



You won't believe it, but I'll tell you anyway. I'm a Level 61 wizard. If you think about it, wands aren't all that hard to use. Point, invoke, joyfully watch your enemy burn in the fires of Reetath. I've watched wizards do it all my life.2



So, when I stumbled across a fairly new-looking wand,3 I picked it up. Wands are fairly generic, right? They all shoot little darts, lightning bolts, fireballs... We've all heard the stories.4 So I wanted to see what I could do! I visited the local library and asked the librarian5 what the word was for "fireball." Then skipped6 to an open field (I wasn't looking to roast a cow or anything), pointed my new wand at some unsuspecting tree, and said Lamboobalar!



The next thing I remember was waking up in my mom's house7 with my hand wrapped up to my elbow and wishing someone would hit me behind the ear with a hammer. It hurt! And what I want to know is, what'd I do wrong?



World Rules



  • Wands are a bit like magical rifles. Fire a bullet through a rifle and the barrel heats up. Fire enough bullets fast enough and the barrel becomes burning hot! No matter what you cast with your wand, the more powerful the spell or the faster you cast the hotter the little honker will get.

  • Magic is the funniest thing. The wand must be held. Not touched by skin necessarily, but held. If you set a wand on a table and scream Wapatay! the wand will happily sit there and ignore you. If you pick up the table, it'll still ignore you. But! grasp it with some chopsticks or with a gauntlet and BOOM! pink mist!8

  • As a bit of a reference point, casting the fireball spell generates enough heat to cause 3rd degree burns on the unprotected hand and wrist. Using the wand to magically rap someone's knuckles, causing them to drop their sword, would only cause noticeable heat if you were paying attention. Casting the dread Sheeeaaaaah-Moogatee-Hah! spell, which consumes your enemy's castle in a somewhat makeshift volcano, will cause an intensely bright light followed immediately by the unprotected wielder converting into a lovely cloud of carbon.9

  • For the purpose of this question, the wand is indestructible. You can't drill through it or cut slots into it, either. The surface has friction similar to any smooth hardwood and does taper from the base to the tip. It's about 1.5cm (5/8") at the base and 0.6cm (1/4") at the tip.

Question Limitations



  • You cannot use magic to solve the problem. Using magic to solve a problem that's a consequence of magic would be like trying to put out a fire with a bucket of gasoline.10 That's why the science-based tag has been applied. No magic!


  • If you're thinking, "this is just a heat-sink problem!" you're exactly right! Good luck!


  • The wand must be usable in a melee situation. In other words, you can't simply encase the wand in a 2-foot diameter column of iron sitting in a custom-designed cart wherein the wizard sits comfortably on a velvet stool while embracing the rod bar. (a) while that would probably work per my rules, it's not practical in a melee situation and (b) the wand would probably blow a hole in the front of the bar, spewing sub-vapor-point iron all over the place. It would look spectacular — and probably fry the wizard anyway. So, the wizard must carry the wand and can't be more encumbered than a suit of plate mail.


  • You may only use 13th century technology (my sincere apologies to TimBII for not stating this here, I can see where it was easy to overlook).



Question: Using 13th century technology, what can I do to protect my erstwhile wizard when he casts a fireball spell with this wand?



Bragging rights to answers that point out how to avoid the cloud of carbon problem — but that's optional.




1Fine! I'm a level 1 wizard... OKAY! I'm some dumb schmuck who found the wand in a gutter! So sue me.



2YES! There are wizards in Grubda! Well there only had to be one! It didn't LOOK that hard! If it means that much to you, ask your own @#*& question! Sheesh, everybody's a critic!



3That body on the sidewalk had nothing to do with the wand. Yes... I'll swear on my mother's grave! Now let me finish!



4Except you! Now shut up!



5You know she's my sister... right?



6Yes, I skipped! Like a 6-year-old girl! I was excited, alright? What would YOU do if you found a wand? Turn it in to the constabulary!? Are you nuts!?



7Don't say it! You were thinking it! Yes you were! I'm saving up for my first horse, alright?



8And molten metal thinly plating everything in a 3-meter radius. Wapatay! is not for the faint of heart.



9Magic should always have a price, don't you think?



10This is obvious, right? Just because it's liquid doesn't mean it'll act like water? Yeah.










share|improve this question











$endgroup$







  • 1




    $begingroup$
    At the time of this comment, no-one seems to have applied a detailed eye to the necessary solution. I'll offer a bounty on this question if it still doesn't get a proper answer after 24 hours. 10 Yep.
    $endgroup$
    – Hoyle's ghost
    6 hours ago







  • 1




    $begingroup$
    Is this supposed to be a question or a puzzle? o_O
    $endgroup$
    – Jesse Amano
    5 hours ago










  • $begingroup$
    @JesseAmano Yeah, I’m pretty sure that this belongs on the Puzzling StackExchange site, given that the OP seems to be playing a game of “read my mind and find the exact solution I’m thinking of” with the answerers.
    $endgroup$
    – nick012000
    4 hours ago










  • $begingroup$
    @JesseAmano Puzzle? How odd? This is a question asking for the application of 13th century technology given the rules of my world. No, it is not a puzzle.
    $endgroup$
    – JBH
    2 hours ago






  • 1




    $begingroup$
    Does this world have dragons?
    $endgroup$
    – Xavon_Wrentaile
    1 hour ago













9












9








9


1



$begingroup$


Backstory



You won't believe it, but I'll tell you anyway. I'm a Level 61 wizard. If you think about it, wands aren't all that hard to use. Point, invoke, joyfully watch your enemy burn in the fires of Reetath. I've watched wizards do it all my life.2



So, when I stumbled across a fairly new-looking wand,3 I picked it up. Wands are fairly generic, right? They all shoot little darts, lightning bolts, fireballs... We've all heard the stories.4 So I wanted to see what I could do! I visited the local library and asked the librarian5 what the word was for "fireball." Then skipped6 to an open field (I wasn't looking to roast a cow or anything), pointed my new wand at some unsuspecting tree, and said Lamboobalar!



The next thing I remember was waking up in my mom's house7 with my hand wrapped up to my elbow and wishing someone would hit me behind the ear with a hammer. It hurt! And what I want to know is, what'd I do wrong?



World Rules



  • Wands are a bit like magical rifles. Fire a bullet through a rifle and the barrel heats up. Fire enough bullets fast enough and the barrel becomes burning hot! No matter what you cast with your wand, the more powerful the spell or the faster you cast the hotter the little honker will get.

  • Magic is the funniest thing. The wand must be held. Not touched by skin necessarily, but held. If you set a wand on a table and scream Wapatay! the wand will happily sit there and ignore you. If you pick up the table, it'll still ignore you. But! grasp it with some chopsticks or with a gauntlet and BOOM! pink mist!8

  • As a bit of a reference point, casting the fireball spell generates enough heat to cause 3rd degree burns on the unprotected hand and wrist. Using the wand to magically rap someone's knuckles, causing them to drop their sword, would only cause noticeable heat if you were paying attention. Casting the dread Sheeeaaaaah-Moogatee-Hah! spell, which consumes your enemy's castle in a somewhat makeshift volcano, will cause an intensely bright light followed immediately by the unprotected wielder converting into a lovely cloud of carbon.9

  • For the purpose of this question, the wand is indestructible. You can't drill through it or cut slots into it, either. The surface has friction similar to any smooth hardwood and does taper from the base to the tip. It's about 1.5cm (5/8") at the base and 0.6cm (1/4") at the tip.

Question Limitations



  • You cannot use magic to solve the problem. Using magic to solve a problem that's a consequence of magic would be like trying to put out a fire with a bucket of gasoline.10 That's why the science-based tag has been applied. No magic!


  • If you're thinking, "this is just a heat-sink problem!" you're exactly right! Good luck!


  • The wand must be usable in a melee situation. In other words, you can't simply encase the wand in a 2-foot diameter column of iron sitting in a custom-designed cart wherein the wizard sits comfortably on a velvet stool while embracing the rod bar. (a) while that would probably work per my rules, it's not practical in a melee situation and (b) the wand would probably blow a hole in the front of the bar, spewing sub-vapor-point iron all over the place. It would look spectacular — and probably fry the wizard anyway. So, the wizard must carry the wand and can't be more encumbered than a suit of plate mail.


  • You may only use 13th century technology (my sincere apologies to TimBII for not stating this here, I can see where it was easy to overlook).



Question: Using 13th century technology, what can I do to protect my erstwhile wizard when he casts a fireball spell with this wand?



Bragging rights to answers that point out how to avoid the cloud of carbon problem — but that's optional.




1Fine! I'm a level 1 wizard... OKAY! I'm some dumb schmuck who found the wand in a gutter! So sue me.



2YES! There are wizards in Grubda! Well there only had to be one! It didn't LOOK that hard! If it means that much to you, ask your own @#*& question! Sheesh, everybody's a critic!



3That body on the sidewalk had nothing to do with the wand. Yes... I'll swear on my mother's grave! Now let me finish!



4Except you! Now shut up!



5You know she's my sister... right?



6Yes, I skipped! Like a 6-year-old girl! I was excited, alright? What would YOU do if you found a wand? Turn it in to the constabulary!? Are you nuts!?



7Don't say it! You were thinking it! Yes you were! I'm saving up for my first horse, alright?



8And molten metal thinly plating everything in a 3-meter radius. Wapatay! is not for the faint of heart.



9Magic should always have a price, don't you think?



10This is obvious, right? Just because it's liquid doesn't mean it'll act like water? Yeah.










share|improve this question











$endgroup$




Backstory



You won't believe it, but I'll tell you anyway. I'm a Level 61 wizard. If you think about it, wands aren't all that hard to use. Point, invoke, joyfully watch your enemy burn in the fires of Reetath. I've watched wizards do it all my life.2



So, when I stumbled across a fairly new-looking wand,3 I picked it up. Wands are fairly generic, right? They all shoot little darts, lightning bolts, fireballs... We've all heard the stories.4 So I wanted to see what I could do! I visited the local library and asked the librarian5 what the word was for "fireball." Then skipped6 to an open field (I wasn't looking to roast a cow or anything), pointed my new wand at some unsuspecting tree, and said Lamboobalar!



The next thing I remember was waking up in my mom's house7 with my hand wrapped up to my elbow and wishing someone would hit me behind the ear with a hammer. It hurt! And what I want to know is, what'd I do wrong?



World Rules



  • Wands are a bit like magical rifles. Fire a bullet through a rifle and the barrel heats up. Fire enough bullets fast enough and the barrel becomes burning hot! No matter what you cast with your wand, the more powerful the spell or the faster you cast the hotter the little honker will get.

  • Magic is the funniest thing. The wand must be held. Not touched by skin necessarily, but held. If you set a wand on a table and scream Wapatay! the wand will happily sit there and ignore you. If you pick up the table, it'll still ignore you. But! grasp it with some chopsticks or with a gauntlet and BOOM! pink mist!8

  • As a bit of a reference point, casting the fireball spell generates enough heat to cause 3rd degree burns on the unprotected hand and wrist. Using the wand to magically rap someone's knuckles, causing them to drop their sword, would only cause noticeable heat if you were paying attention. Casting the dread Sheeeaaaaah-Moogatee-Hah! spell, which consumes your enemy's castle in a somewhat makeshift volcano, will cause an intensely bright light followed immediately by the unprotected wielder converting into a lovely cloud of carbon.9

  • For the purpose of this question, the wand is indestructible. You can't drill through it or cut slots into it, either. The surface has friction similar to any smooth hardwood and does taper from the base to the tip. It's about 1.5cm (5/8") at the base and 0.6cm (1/4") at the tip.

Question Limitations



  • You cannot use magic to solve the problem. Using magic to solve a problem that's a consequence of magic would be like trying to put out a fire with a bucket of gasoline.10 That's why the science-based tag has been applied. No magic!


  • If you're thinking, "this is just a heat-sink problem!" you're exactly right! Good luck!


  • The wand must be usable in a melee situation. In other words, you can't simply encase the wand in a 2-foot diameter column of iron sitting in a custom-designed cart wherein the wizard sits comfortably on a velvet stool while embracing the rod bar. (a) while that would probably work per my rules, it's not practical in a melee situation and (b) the wand would probably blow a hole in the front of the bar, spewing sub-vapor-point iron all over the place. It would look spectacular — and probably fry the wizard anyway. So, the wizard must carry the wand and can't be more encumbered than a suit of plate mail.


  • You may only use 13th century technology (my sincere apologies to TimBII for not stating this here, I can see where it was easy to overlook).



Question: Using 13th century technology, what can I do to protect my erstwhile wizard when he casts a fireball spell with this wand?



Bragging rights to answers that point out how to avoid the cloud of carbon problem — but that's optional.




1Fine! I'm a level 1 wizard... OKAY! I'm some dumb schmuck who found the wand in a gutter! So sue me.



2YES! There are wizards in Grubda! Well there only had to be one! It didn't LOOK that hard! If it means that much to you, ask your own @#*& question! Sheesh, everybody's a critic!



3That body on the sidewalk had nothing to do with the wand. Yes... I'll swear on my mother's grave! Now let me finish!



4Except you! Now shut up!



5You know she's my sister... right?



6Yes, I skipped! Like a 6-year-old girl! I was excited, alright? What would YOU do if you found a wand? Turn it in to the constabulary!? Are you nuts!?



7Don't say it! You were thinking it! Yes you were! I'm saving up for my first horse, alright?



8And molten metal thinly plating everything in a 3-meter radius. Wapatay! is not for the faint of heart.



9Magic should always have a price, don't you think?



10This is obvious, right? Just because it's liquid doesn't mean it'll act like water? Yeah.







science-based protection thermodynamics






share|improve this question















share|improve this question













share|improve this question




share|improve this question








edited 58 mins ago







JBH

















asked 7 hours ago









JBHJBH

51.1k7105247




51.1k7105247







  • 1




    $begingroup$
    At the time of this comment, no-one seems to have applied a detailed eye to the necessary solution. I'll offer a bounty on this question if it still doesn't get a proper answer after 24 hours. 10 Yep.
    $endgroup$
    – Hoyle's ghost
    6 hours ago







  • 1




    $begingroup$
    Is this supposed to be a question or a puzzle? o_O
    $endgroup$
    – Jesse Amano
    5 hours ago










  • $begingroup$
    @JesseAmano Yeah, I’m pretty sure that this belongs on the Puzzling StackExchange site, given that the OP seems to be playing a game of “read my mind and find the exact solution I’m thinking of” with the answerers.
    $endgroup$
    – nick012000
    4 hours ago










  • $begingroup$
    @JesseAmano Puzzle? How odd? This is a question asking for the application of 13th century technology given the rules of my world. No, it is not a puzzle.
    $endgroup$
    – JBH
    2 hours ago






  • 1




    $begingroup$
    Does this world have dragons?
    $endgroup$
    – Xavon_Wrentaile
    1 hour ago












  • 1




    $begingroup$
    At the time of this comment, no-one seems to have applied a detailed eye to the necessary solution. I'll offer a bounty on this question if it still doesn't get a proper answer after 24 hours. 10 Yep.
    $endgroup$
    – Hoyle's ghost
    6 hours ago







  • 1




    $begingroup$
    Is this supposed to be a question or a puzzle? o_O
    $endgroup$
    – Jesse Amano
    5 hours ago










  • $begingroup$
    @JesseAmano Yeah, I’m pretty sure that this belongs on the Puzzling StackExchange site, given that the OP seems to be playing a game of “read my mind and find the exact solution I’m thinking of” with the answerers.
    $endgroup$
    – nick012000
    4 hours ago










  • $begingroup$
    @JesseAmano Puzzle? How odd? This is a question asking for the application of 13th century technology given the rules of my world. No, it is not a puzzle.
    $endgroup$
    – JBH
    2 hours ago






  • 1




    $begingroup$
    Does this world have dragons?
    $endgroup$
    – Xavon_Wrentaile
    1 hour ago







1




1




$begingroup$
At the time of this comment, no-one seems to have applied a detailed eye to the necessary solution. I'll offer a bounty on this question if it still doesn't get a proper answer after 24 hours. 10 Yep.
$endgroup$
– Hoyle's ghost
6 hours ago





$begingroup$
At the time of this comment, no-one seems to have applied a detailed eye to the necessary solution. I'll offer a bounty on this question if it still doesn't get a proper answer after 24 hours. 10 Yep.
$endgroup$
– Hoyle's ghost
6 hours ago





1




1




$begingroup$
Is this supposed to be a question or a puzzle? o_O
$endgroup$
– Jesse Amano
5 hours ago




$begingroup$
Is this supposed to be a question or a puzzle? o_O
$endgroup$
– Jesse Amano
5 hours ago












$begingroup$
@JesseAmano Yeah, I’m pretty sure that this belongs on the Puzzling StackExchange site, given that the OP seems to be playing a game of “read my mind and find the exact solution I’m thinking of” with the answerers.
$endgroup$
– nick012000
4 hours ago




$begingroup$
@JesseAmano Yeah, I’m pretty sure that this belongs on the Puzzling StackExchange site, given that the OP seems to be playing a game of “read my mind and find the exact solution I’m thinking of” with the answerers.
$endgroup$
– nick012000
4 hours ago












$begingroup$
@JesseAmano Puzzle? How odd? This is a question asking for the application of 13th century technology given the rules of my world. No, it is not a puzzle.
$endgroup$
– JBH
2 hours ago




$begingroup$
@JesseAmano Puzzle? How odd? This is a question asking for the application of 13th century technology given the rules of my world. No, it is not a puzzle.
$endgroup$
– JBH
2 hours ago




1




1




$begingroup$
Does this world have dragons?
$endgroup$
– Xavon_Wrentaile
1 hour ago




$begingroup$
Does this world have dragons?
$endgroup$
– Xavon_Wrentaile
1 hour ago










6 Answers
6






active

oldest

votes


















6












$begingroup$

The solution to your magic wand problem is actually a staff... I love when a dumb trope is the right solution to another. Much like your chopsticks proposal, you are using a device to hold it away from you. The staff puts distance between your hand and the wand which is important when you dealing with limited insulating materials. While the heat of the wand is enough to cause 3rd degree burns, the head of the staff will heat up a lot at the end but much much less at the handle; so, slip the wand into the end of a staff and it's like holding a pot over a flame.



An ideal staff would probably look a lot like the diagram below. The wand itself would be inserted into a glass or high-cone ceramic since these are the most heat resistant materials available in the middle ages. The holster would then be held by a steel head, preferably with seamingly decorative spines loops or other patterns to help it dissipate heat. Then the head would tapper back, thick to thin. The steel closest to the head will get hottest so extra girth will help keep it from melting or warping when it reaches malleable temperatures, but the tapper makes sure that less heat can spread back through the shaft. Then the handle itself would be wooden. Since wood does not propagate heat well, this will further insulate you.



enter image description here



Your castle volcano problem is a bit ridiculous for which you will need a ridiculous staff (aka pike staff), that will put a great as possible distance between caster and wand. More importantly, you need what basically amounts to a ballistic shelter similar to what Davy Crockett crews often used. That much heat will be explosive, and you will be in the blast radius, but if you are protected by a bunker where you are physically underground, the blast will mostly pass over you. The pike staff will be destroyed, but the caster, ducking under the level of the ground and with a shelter to absorb most of the radiant heat might just survive the blast.



enter image description here






share|improve this answer











$endgroup$












  • $begingroup$
    Nope, unless I'm much mistaken (never happens BTW.) then your answer relies on a mistaken premise.
    $endgroup$
    – Hoyle's ghost
    4 hours ago











  • $begingroup$
    "But! grasp it with some chopsticks or with a gauntlet and BOOM! pink mist!" This implies that any device you use to hold it counts as grasping it; so, even the most fickle interpretation of this is that your staff might need a grasping lever, but the basic solution remains.
    $endgroup$
    – Nosajimiki
    3 hours ago



















2












$begingroup$

Treat the wand like it came from a blacksmith's forge



Gloved blacksmith



While some blacksmiths don't use gloves at the forge, others do. And they generally wear them for very short exposures to heat (a wand going whamo should count).



Leather gloves won't be enough to protect against very high heat (fireball spells) but it will protect you from radiant heat near the wand. You also want a leather apron, some kind of safety glasses, and boots.




About the Leather gloves - Experienced blacksmiths will sometimes
recommend no gloves. I strongly recommend a glove on your not-hammer
hand. At least just to start. This will prevent burns. Later, as you
get more experienced with blacksmithing you can decide what you like.



About the boots - This is important because you are going to be
dropping hot pieces of steel and you want a solid pair of boots to
protect your feet from burns. If you plan on handling heavy pieces of
iron and steel then I would go with steel-toed shoes.




So lots of safety gear to start then you can reduce it when you get used to spellcasting.



This site also recommends wolf jaw tongs (not plain jaw/flat). I'd recommend some made special to firmly hold your wand near the base. If blacksmiths can use these to hold on to heavy metal glowing like a miniature sun, and then be able to manipulate the metal on the anvil, you can use them to hold your freaking wand still while you cast.



Remember, the apron and boots and eye coverings (if possible) are for when you are an idiot and drop the hot wand as soon as something blasts out of it.



Tongs



Okay, so this covers you for basic fireballs. Now, what about the Sheeeaaaaah-Moogatee-Hah! spell?



Okay, this requires some setup. I recommend the following:



  • A large barrel made of thick oak plated with metal with a 3-4 inch hole drilled near the top.

  • Fill with water such that it will later reach up to the bottom of the hole.

  • Get in the barrel with something protective on your head.

  • Put on a leather glove.

  • Stick your blacksmithing tongs out the hole and grab the wand.

  • Make sure that under the wand is a bucket of water or a pile of sand.

  • Cast the spell.

  • Drop the wand.

Alternatively...fire that sucker from a stone castle window or rampart, the kind archers shoot out of. Have a trusted running retrieve the wand after it drops in the water/sand and is cool enough to pick up with tongs or a gloved hand.



No, it's not melee-worthy like your fireball and other low to high level spells are. But if you want the nuclear option, you need to build a launch point. This completely violates your requirements, but seriously, do soldiers with fire launchers run around with them in combat? No, they use a sturdy base, sometimes one that is somewhat portable. If you fight with a cannon can you carry it in your pocket then pull it out and boom? Nope.



Remember, your question was protection for a fireball spell. And you state: "casting the fireball spell generates enough heat to cause 3rd degree burns on the unprotected hand and wrist." My solution easily protects against that.






share|improve this answer











$endgroup$












  • $begingroup$
    Again, partially correct only.
    $endgroup$
    – Hoyle's ghost
    4 hours ago










  • $begingroup$
    @Hoyle'sghost Well write an answer already!
    $endgroup$
    – Cyn
    4 hours ago










  • $begingroup$
    I yet may, though I much prefer to watch others set themselves up for a fall. :)
    $endgroup$
    – Hoyle's ghost
    4 hours ago










  • $begingroup$
    @Hoyle'sghost Ha ha I can tell.
    $endgroup$
    – Cyn
    4 hours ago






  • 1




    $begingroup$
    I think he means because tongs are not exactly wieldy in a melee, but I do think you make a lot of good points about general blacksmith safety precautions.
    $endgroup$
    – Nosajimiki
    2 hours ago


















2












$begingroup$

Do what every successful business has ever done with hard to crack problems: outsource it.



In order to outsource the problem of hot wands, you will need:



  • feathers

  • a bow

See a picture below of some witches invoking some explosive fireballs (you know, the kinda that goes boom when it hits something). Notice that they are grasping the wand firmly with the right hand, while grasping the bow with the left hand.



Volley!



Once the wand flies, the problem has been successfully outsourced to whomever you were aiming your fireball at. Any heating concerns are now SEP's (Someone Else's Problem).



And before you tell me that goes against the melee usage rule: you can use an arrow on a melee fight if you really want.






share|improve this answer











$endgroup$








  • 1




    $begingroup$
    Not sure how this would be any better than just dropping it quickly. But a slight variation to this: fire the arrow, and when then wounded person grabs the shaft in reaction to being shot, call out the enchantment blowing up him, and everyone around him. IE: make your enemy grasp the wand
    $endgroup$
    – Nosajimiki
    2 hours ago










  • $begingroup$
    "Not sure how this would be any better than just dropping it quickly." Unless you are dropping the wand from a very high place, this method is different because it also adds maiming to arson.
    $endgroup$
    – Renan
    2 hours ago











  • $begingroup$
    This is an awesome answer, but I find myself wondering about the conductive properties of the wand itself. Ideally, you want your wand to be made of a material that doesn't disperse the heat to the outer surface of the wand for at least a quarter of a second after the spell is cast. Oh, and you've just invented 'Wandcraft' as a surname right alongside 'Fletcher' in the 13th century by turning the manufacture of wands into an industry. Other than that though let the destruction commence!
    $endgroup$
    – Tim B II
    1 hour ago






  • 1




    $begingroup$
    @TimBII You're right. Maybe there is some resin or varnish that could be used around the shaft to keep the heat in; This is not my forte, so I would outsource (and no jokes involved this time) this problem to the local alchemist.
    $endgroup$
    – Renan
    1 hour ago


















1












$begingroup$

Freeze it



Before using the wand, freeze it. In a combat situation, this will involve frequent wand switches. Take a tub of ice with you. Store all your wands in the ice, so they are nice and cold. Put on a good pair of gloves. Reach into the tub and grab a wand. Aim and shout the appropriate spell. Immediately drop the wand back into the ice. If your hand is overheated, stick it in the ice. Repeat with the next wand when your hand starts getting cold.



In the thirteenth century your source for ice would be the top of a mountain. Take a hay covered wagon up the mountain to where it gets cold. Make a hole in the center of the hay. Put ice in the hole. Lots of ice. Cover it up with hay from the sides. By the time you get to the bottom of the mountain, you'll have less ice.



Stuff the wands in the ice at any time, even while still at the top of the mountain. You want them to get as cold as possible so that you can maximize the safe power of the spell.






share|improve this answer









$endgroup$












  • $begingroup$
    This appears to be a bit of a problem for Question Limitations, bullet #3, the solution must be practical in a melee situation. But it's a clever idea!
    $endgroup$
    – JBH
    2 hours ago










  • $begingroup$
    Didn't the narrator just "happen to find" a single wand?
    $endgroup$
    – Cyn
    1 hour ago


















1












$begingroup$

I have an answer, but I'm not sure if it breaks the rules or not it goes against the rules on two fronts - drilling holes in the wand (thanks JBH) and violates the 13th century rules (thanks Nosajimiki). But, I'm going to throw it out there anyway as an apocryphal answer.



First of all, you're completely right; this is a classic heat sink problem. But, you're also wrong; a gun barrel heats up because of convection from the gases in the gun barrel; the barrel itself is metal for strength, and also because you actually want the barrel to get hot so that the next bullet doesn't discharge prematurely because of a massive heat buildup in the rifle. This was the whole point of gatling guns back in the day; give each barrel time to cool down before being used to fire another bullet.



In point of fact, the whole problem you have with your wand in terms of thermodynamics is that it isn't like a gun barrel, which would make better sense. What you really want to do is conduct the heat away quickly so that it an dissipate in the atmosphere rather than cooking your hand. This is also why gun stocks and handles are not made of metal - They're designed out of wood, pearl, composites, etc. to shield your hand from the excess heat buildup.



I'm going to assume that your wand is some form of magical wood or other element that traps the heat to insane levels without spontaneously combusting. That is about the only model which could work the way you describe, although it also means that the biggest issue with your wand isn't that it gets hot, it's that it doesn't cool down fast enough. That said, the solution is twofold; firstly you need to reinvent a 'stock', or wand handle, and then you need to borrow an ammunition concept from science fiction.



Put more simply, you need to insulate the hand AND absorb the heat out of the wand fast enough so as not to overwhelm the insulative properties of the stock which you use to grip.



Let's deal with the stock first. If it wasn't for their combustible nature, wood or paper would actually make a good option here. But, in this instance, I'm going to suggest a fibrelass handle with a styrofoam core. This would have several benefits;



. You can turn this into a simple collar or tube which your wand slides into

. The styrofoam will grip the wand quite well, meaning less chance of it slipping out in a fight

. Easily replaceable if the styrofoam starts to wear.



Secondly, we need to create something like a thermal clip for your wand. Basically, the idea of a thermal clip in scifi games is that we have all moved to energy weapons, but those weapons can't disperse their heat so they capture it in a disposable thermal clip, which basically stores the heat and can be replaced quickly in battle. In gaming the idea is to keep the idea of ammunition. In your scenario, it's actually a pretty good analogue for the problem you face.



I'm going to suggest that you drill a small hole down the centre line of your wand (hence the idea that it could be outside the rules) and put a small rod of something like Tungsten in it, with a screw thread on the back end. Why tungsten? Because it has the highest melting point but still pretty good thermal conduction properties. Then what you need is a supply of Liquid N2 bulbs that screw or clip on at the base of the wand.



The idea would be that the tungsten conducts the heat to the bulb, which absorbs the heat and converts the nitrogen back to gas. Once it has done that to all the nitrogen in the bulb, you discard and replace.



Now, I know this isn't a perfect solution; how do you keep your N2 bulbs cold in the field? How many shots can a nitrogen bulb acting as a thermal clip absorb? Seriously, you want me to insert what in my wand? How is that going to affect the magical properties?



I suggest that you get one of your friends who wants to be a 'level 6' magician to assist you with some field trials (preferably one you don't like very much) but ultimately, the engineering of this is quite simple - you need to dissipate heat quickly while insulating your hand from it until the heat has dissipated.



For that reason, gun barrels getting hot is actually a good thing; it's touching them that's bad. Your wand needs a similar dynamic. Putting a styrofoam / fibreglass grip around it will hopefully protect your hand while the 'thermal clip' sucks as much of the excess heat out as quickly as it can, reducing the load on the insulated grip.



In any case, be very careful during your testing and don't let anyone do this who will be indispensable in your future magical career.






share|improve this answer











$endgroup$












  • $begingroup$
    Partially correct only, I'm afraid.
    $endgroup$
    – Hoyle's ghost
    4 hours ago










  • $begingroup$
    @Hoyle'sghost So, which part do you disagree with?
    $endgroup$
    – Tim B II
    4 hours ago










  • $begingroup$
    Several of these materials such as styrofoam, fiberglass, tungsten, and liquid nitrogen violate the 13th century requirement.
    $endgroup$
    – Nosajimiki
    2 hours ago











  • $begingroup$
    You're right, drilling a hole in the wand violates World Rules, bullet #4. However, it does make for a fascinating Steampunk idea.
    $endgroup$
    – JBH
    2 hours ago










  • $begingroup$
    @Nosajimiki - Ha! You're right, I actually went looking for tech level requirements (they're notorious in our questions) and didn't see them in the rules. I should've looked in the actual question... I'll edit up top to reflect this is an apocryphal answer
    $endgroup$
    – Tim B II
    2 hours ago


















0












$begingroup$

Sand/Water Cooling



The Heat dissipation doesn't need to be a function of the wand itself. If the user's hands are themselves insulated, they can use traditional cooling techniques. A wand user can carry classical cooling systems as part of their kit. Since I presume this wand doesn't need ammunition, if you equip them similar to a medieval fusilier they can carry sand cartridges/water pouches instead of ammo






share|improve this answer









$endgroup$












  • $begingroup$
    Insulation, yes, partial answer though.
    $endgroup$
    – Hoyle's ghost
    4 hours ago










  • $begingroup$
    This is an interesting idea. How do you use the sand and/or water? Can you add a bit of detail?
    $endgroup$
    – JBH
    2 hours ago











Your Answer








StackExchange.ready(function()
var channelOptions =
tags: "".split(" "),
id: "579"
;
initTagRenderer("".split(" "), "".split(" "), channelOptions);

StackExchange.using("externalEditor", function()
// Have to fire editor after snippets, if snippets enabled
if (StackExchange.settings.snippets.snippetsEnabled)
StackExchange.using("snippets", function()
createEditor();
);

else
createEditor();

);

function createEditor()
StackExchange.prepareEditor(
heartbeatType: 'answer',
autoActivateHeartbeat: false,
convertImagesToLinks: false,
noModals: true,
showLowRepImageUploadWarning: true,
reputationToPostImages: null,
bindNavPrevention: true,
postfix: "",
imageUploader:
brandingHtml: "Powered by u003ca class="icon-imgur-white" href="https://imgur.com/"u003eu003c/au003e",
contentPolicyHtml: "User contributions licensed under u003ca href="https://creativecommons.org/licenses/by-sa/3.0/"u003ecc by-sa 3.0 with attribution requiredu003c/au003e u003ca href="https://stackoverflow.com/legal/content-policy"u003e(content policy)u003c/au003e",
allowUrls: true
,
noCode: true, onDemand: true,
discardSelector: ".discard-answer"
,immediatelyShowMarkdownHelp:true
);



);













draft saved

draft discarded


















StackExchange.ready(
function ()
StackExchange.openid.initPostLogin('.new-post-login', 'https%3a%2f%2fworldbuilding.stackexchange.com%2fquestions%2f147181%2fmanaging-heat-dissipation-in-a-magic-wand%23new-answer', 'question_page');

);

Post as a guest















Required, but never shown

























6 Answers
6






active

oldest

votes








6 Answers
6






active

oldest

votes









active

oldest

votes






active

oldest

votes









6












$begingroup$

The solution to your magic wand problem is actually a staff... I love when a dumb trope is the right solution to another. Much like your chopsticks proposal, you are using a device to hold it away from you. The staff puts distance between your hand and the wand which is important when you dealing with limited insulating materials. While the heat of the wand is enough to cause 3rd degree burns, the head of the staff will heat up a lot at the end but much much less at the handle; so, slip the wand into the end of a staff and it's like holding a pot over a flame.



An ideal staff would probably look a lot like the diagram below. The wand itself would be inserted into a glass or high-cone ceramic since these are the most heat resistant materials available in the middle ages. The holster would then be held by a steel head, preferably with seamingly decorative spines loops or other patterns to help it dissipate heat. Then the head would tapper back, thick to thin. The steel closest to the head will get hottest so extra girth will help keep it from melting or warping when it reaches malleable temperatures, but the tapper makes sure that less heat can spread back through the shaft. Then the handle itself would be wooden. Since wood does not propagate heat well, this will further insulate you.



enter image description here



Your castle volcano problem is a bit ridiculous for which you will need a ridiculous staff (aka pike staff), that will put a great as possible distance between caster and wand. More importantly, you need what basically amounts to a ballistic shelter similar to what Davy Crockett crews often used. That much heat will be explosive, and you will be in the blast radius, but if you are protected by a bunker where you are physically underground, the blast will mostly pass over you. The pike staff will be destroyed, but the caster, ducking under the level of the ground and with a shelter to absorb most of the radiant heat might just survive the blast.



enter image description here






share|improve this answer











$endgroup$












  • $begingroup$
    Nope, unless I'm much mistaken (never happens BTW.) then your answer relies on a mistaken premise.
    $endgroup$
    – Hoyle's ghost
    4 hours ago











  • $begingroup$
    "But! grasp it with some chopsticks or with a gauntlet and BOOM! pink mist!" This implies that any device you use to hold it counts as grasping it; so, even the most fickle interpretation of this is that your staff might need a grasping lever, but the basic solution remains.
    $endgroup$
    – Nosajimiki
    3 hours ago
















6












$begingroup$

The solution to your magic wand problem is actually a staff... I love when a dumb trope is the right solution to another. Much like your chopsticks proposal, you are using a device to hold it away from you. The staff puts distance between your hand and the wand which is important when you dealing with limited insulating materials. While the heat of the wand is enough to cause 3rd degree burns, the head of the staff will heat up a lot at the end but much much less at the handle; so, slip the wand into the end of a staff and it's like holding a pot over a flame.



An ideal staff would probably look a lot like the diagram below. The wand itself would be inserted into a glass or high-cone ceramic since these are the most heat resistant materials available in the middle ages. The holster would then be held by a steel head, preferably with seamingly decorative spines loops or other patterns to help it dissipate heat. Then the head would tapper back, thick to thin. The steel closest to the head will get hottest so extra girth will help keep it from melting or warping when it reaches malleable temperatures, but the tapper makes sure that less heat can spread back through the shaft. Then the handle itself would be wooden. Since wood does not propagate heat well, this will further insulate you.



enter image description here



Your castle volcano problem is a bit ridiculous for which you will need a ridiculous staff (aka pike staff), that will put a great as possible distance between caster and wand. More importantly, you need what basically amounts to a ballistic shelter similar to what Davy Crockett crews often used. That much heat will be explosive, and you will be in the blast radius, but if you are protected by a bunker where you are physically underground, the blast will mostly pass over you. The pike staff will be destroyed, but the caster, ducking under the level of the ground and with a shelter to absorb most of the radiant heat might just survive the blast.



enter image description here






share|improve this answer











$endgroup$












  • $begingroup$
    Nope, unless I'm much mistaken (never happens BTW.) then your answer relies on a mistaken premise.
    $endgroup$
    – Hoyle's ghost
    4 hours ago











  • $begingroup$
    "But! grasp it with some chopsticks or with a gauntlet and BOOM! pink mist!" This implies that any device you use to hold it counts as grasping it; so, even the most fickle interpretation of this is that your staff might need a grasping lever, but the basic solution remains.
    $endgroup$
    – Nosajimiki
    3 hours ago














6












6








6





$begingroup$

The solution to your magic wand problem is actually a staff... I love when a dumb trope is the right solution to another. Much like your chopsticks proposal, you are using a device to hold it away from you. The staff puts distance between your hand and the wand which is important when you dealing with limited insulating materials. While the heat of the wand is enough to cause 3rd degree burns, the head of the staff will heat up a lot at the end but much much less at the handle; so, slip the wand into the end of a staff and it's like holding a pot over a flame.



An ideal staff would probably look a lot like the diagram below. The wand itself would be inserted into a glass or high-cone ceramic since these are the most heat resistant materials available in the middle ages. The holster would then be held by a steel head, preferably with seamingly decorative spines loops or other patterns to help it dissipate heat. Then the head would tapper back, thick to thin. The steel closest to the head will get hottest so extra girth will help keep it from melting or warping when it reaches malleable temperatures, but the tapper makes sure that less heat can spread back through the shaft. Then the handle itself would be wooden. Since wood does not propagate heat well, this will further insulate you.



enter image description here



Your castle volcano problem is a bit ridiculous for which you will need a ridiculous staff (aka pike staff), that will put a great as possible distance between caster and wand. More importantly, you need what basically amounts to a ballistic shelter similar to what Davy Crockett crews often used. That much heat will be explosive, and you will be in the blast radius, but if you are protected by a bunker where you are physically underground, the blast will mostly pass over you. The pike staff will be destroyed, but the caster, ducking under the level of the ground and with a shelter to absorb most of the radiant heat might just survive the blast.



enter image description here






share|improve this answer











$endgroup$



The solution to your magic wand problem is actually a staff... I love when a dumb trope is the right solution to another. Much like your chopsticks proposal, you are using a device to hold it away from you. The staff puts distance between your hand and the wand which is important when you dealing with limited insulating materials. While the heat of the wand is enough to cause 3rd degree burns, the head of the staff will heat up a lot at the end but much much less at the handle; so, slip the wand into the end of a staff and it's like holding a pot over a flame.



An ideal staff would probably look a lot like the diagram below. The wand itself would be inserted into a glass or high-cone ceramic since these are the most heat resistant materials available in the middle ages. The holster would then be held by a steel head, preferably with seamingly decorative spines loops or other patterns to help it dissipate heat. Then the head would tapper back, thick to thin. The steel closest to the head will get hottest so extra girth will help keep it from melting or warping when it reaches malleable temperatures, but the tapper makes sure that less heat can spread back through the shaft. Then the handle itself would be wooden. Since wood does not propagate heat well, this will further insulate you.



enter image description here



Your castle volcano problem is a bit ridiculous for which you will need a ridiculous staff (aka pike staff), that will put a great as possible distance between caster and wand. More importantly, you need what basically amounts to a ballistic shelter similar to what Davy Crockett crews often used. That much heat will be explosive, and you will be in the blast radius, but if you are protected by a bunker where you are physically underground, the blast will mostly pass over you. The pike staff will be destroyed, but the caster, ducking under the level of the ground and with a shelter to absorb most of the radiant heat might just survive the blast.



enter image description here







share|improve this answer














share|improve this answer



share|improve this answer








edited 2 hours ago

























answered 7 hours ago









NosajimikiNosajimiki

4,5271231




4,5271231











  • $begingroup$
    Nope, unless I'm much mistaken (never happens BTW.) then your answer relies on a mistaken premise.
    $endgroup$
    – Hoyle's ghost
    4 hours ago











  • $begingroup$
    "But! grasp it with some chopsticks or with a gauntlet and BOOM! pink mist!" This implies that any device you use to hold it counts as grasping it; so, even the most fickle interpretation of this is that your staff might need a grasping lever, but the basic solution remains.
    $endgroup$
    – Nosajimiki
    3 hours ago

















  • $begingroup$
    Nope, unless I'm much mistaken (never happens BTW.) then your answer relies on a mistaken premise.
    $endgroup$
    – Hoyle's ghost
    4 hours ago











  • $begingroup$
    "But! grasp it with some chopsticks or with a gauntlet and BOOM! pink mist!" This implies that any device you use to hold it counts as grasping it; so, even the most fickle interpretation of this is that your staff might need a grasping lever, but the basic solution remains.
    $endgroup$
    – Nosajimiki
    3 hours ago
















$begingroup$
Nope, unless I'm much mistaken (never happens BTW.) then your answer relies on a mistaken premise.
$endgroup$
– Hoyle's ghost
4 hours ago





$begingroup$
Nope, unless I'm much mistaken (never happens BTW.) then your answer relies on a mistaken premise.
$endgroup$
– Hoyle's ghost
4 hours ago













$begingroup$
"But! grasp it with some chopsticks or with a gauntlet and BOOM! pink mist!" This implies that any device you use to hold it counts as grasping it; so, even the most fickle interpretation of this is that your staff might need a grasping lever, but the basic solution remains.
$endgroup$
– Nosajimiki
3 hours ago





$begingroup$
"But! grasp it with some chopsticks or with a gauntlet and BOOM! pink mist!" This implies that any device you use to hold it counts as grasping it; so, even the most fickle interpretation of this is that your staff might need a grasping lever, but the basic solution remains.
$endgroup$
– Nosajimiki
3 hours ago












2












$begingroup$

Treat the wand like it came from a blacksmith's forge



Gloved blacksmith



While some blacksmiths don't use gloves at the forge, others do. And they generally wear them for very short exposures to heat (a wand going whamo should count).



Leather gloves won't be enough to protect against very high heat (fireball spells) but it will protect you from radiant heat near the wand. You also want a leather apron, some kind of safety glasses, and boots.




About the Leather gloves - Experienced blacksmiths will sometimes
recommend no gloves. I strongly recommend a glove on your not-hammer
hand. At least just to start. This will prevent burns. Later, as you
get more experienced with blacksmithing you can decide what you like.



About the boots - This is important because you are going to be
dropping hot pieces of steel and you want a solid pair of boots to
protect your feet from burns. If you plan on handling heavy pieces of
iron and steel then I would go with steel-toed shoes.




So lots of safety gear to start then you can reduce it when you get used to spellcasting.



This site also recommends wolf jaw tongs (not plain jaw/flat). I'd recommend some made special to firmly hold your wand near the base. If blacksmiths can use these to hold on to heavy metal glowing like a miniature sun, and then be able to manipulate the metal on the anvil, you can use them to hold your freaking wand still while you cast.



Remember, the apron and boots and eye coverings (if possible) are for when you are an idiot and drop the hot wand as soon as something blasts out of it.



Tongs



Okay, so this covers you for basic fireballs. Now, what about the Sheeeaaaaah-Moogatee-Hah! spell?



Okay, this requires some setup. I recommend the following:



  • A large barrel made of thick oak plated with metal with a 3-4 inch hole drilled near the top.

  • Fill with water such that it will later reach up to the bottom of the hole.

  • Get in the barrel with something protective on your head.

  • Put on a leather glove.

  • Stick your blacksmithing tongs out the hole and grab the wand.

  • Make sure that under the wand is a bucket of water or a pile of sand.

  • Cast the spell.

  • Drop the wand.

Alternatively...fire that sucker from a stone castle window or rampart, the kind archers shoot out of. Have a trusted running retrieve the wand after it drops in the water/sand and is cool enough to pick up with tongs or a gloved hand.



No, it's not melee-worthy like your fireball and other low to high level spells are. But if you want the nuclear option, you need to build a launch point. This completely violates your requirements, but seriously, do soldiers with fire launchers run around with them in combat? No, they use a sturdy base, sometimes one that is somewhat portable. If you fight with a cannon can you carry it in your pocket then pull it out and boom? Nope.



Remember, your question was protection for a fireball spell. And you state: "casting the fireball spell generates enough heat to cause 3rd degree burns on the unprotected hand and wrist." My solution easily protects against that.






share|improve this answer











$endgroup$












  • $begingroup$
    Again, partially correct only.
    $endgroup$
    – Hoyle's ghost
    4 hours ago










  • $begingroup$
    @Hoyle'sghost Well write an answer already!
    $endgroup$
    – Cyn
    4 hours ago










  • $begingroup$
    I yet may, though I much prefer to watch others set themselves up for a fall. :)
    $endgroup$
    – Hoyle's ghost
    4 hours ago










  • $begingroup$
    @Hoyle'sghost Ha ha I can tell.
    $endgroup$
    – Cyn
    4 hours ago






  • 1




    $begingroup$
    I think he means because tongs are not exactly wieldy in a melee, but I do think you make a lot of good points about general blacksmith safety precautions.
    $endgroup$
    – Nosajimiki
    2 hours ago















2












$begingroup$

Treat the wand like it came from a blacksmith's forge



Gloved blacksmith



While some blacksmiths don't use gloves at the forge, others do. And they generally wear them for very short exposures to heat (a wand going whamo should count).



Leather gloves won't be enough to protect against very high heat (fireball spells) but it will protect you from radiant heat near the wand. You also want a leather apron, some kind of safety glasses, and boots.




About the Leather gloves - Experienced blacksmiths will sometimes
recommend no gloves. I strongly recommend a glove on your not-hammer
hand. At least just to start. This will prevent burns. Later, as you
get more experienced with blacksmithing you can decide what you like.



About the boots - This is important because you are going to be
dropping hot pieces of steel and you want a solid pair of boots to
protect your feet from burns. If you plan on handling heavy pieces of
iron and steel then I would go with steel-toed shoes.




So lots of safety gear to start then you can reduce it when you get used to spellcasting.



This site also recommends wolf jaw tongs (not plain jaw/flat). I'd recommend some made special to firmly hold your wand near the base. If blacksmiths can use these to hold on to heavy metal glowing like a miniature sun, and then be able to manipulate the metal on the anvil, you can use them to hold your freaking wand still while you cast.



Remember, the apron and boots and eye coverings (if possible) are for when you are an idiot and drop the hot wand as soon as something blasts out of it.



Tongs



Okay, so this covers you for basic fireballs. Now, what about the Sheeeaaaaah-Moogatee-Hah! spell?



Okay, this requires some setup. I recommend the following:



  • A large barrel made of thick oak plated with metal with a 3-4 inch hole drilled near the top.

  • Fill with water such that it will later reach up to the bottom of the hole.

  • Get in the barrel with something protective on your head.

  • Put on a leather glove.

  • Stick your blacksmithing tongs out the hole and grab the wand.

  • Make sure that under the wand is a bucket of water or a pile of sand.

  • Cast the spell.

  • Drop the wand.

Alternatively...fire that sucker from a stone castle window or rampart, the kind archers shoot out of. Have a trusted running retrieve the wand after it drops in the water/sand and is cool enough to pick up with tongs or a gloved hand.



No, it's not melee-worthy like your fireball and other low to high level spells are. But if you want the nuclear option, you need to build a launch point. This completely violates your requirements, but seriously, do soldiers with fire launchers run around with them in combat? No, they use a sturdy base, sometimes one that is somewhat portable. If you fight with a cannon can you carry it in your pocket then pull it out and boom? Nope.



Remember, your question was protection for a fireball spell. And you state: "casting the fireball spell generates enough heat to cause 3rd degree burns on the unprotected hand and wrist." My solution easily protects against that.






share|improve this answer











$endgroup$












  • $begingroup$
    Again, partially correct only.
    $endgroup$
    – Hoyle's ghost
    4 hours ago










  • $begingroup$
    @Hoyle'sghost Well write an answer already!
    $endgroup$
    – Cyn
    4 hours ago










  • $begingroup$
    I yet may, though I much prefer to watch others set themselves up for a fall. :)
    $endgroup$
    – Hoyle's ghost
    4 hours ago










  • $begingroup$
    @Hoyle'sghost Ha ha I can tell.
    $endgroup$
    – Cyn
    4 hours ago






  • 1




    $begingroup$
    I think he means because tongs are not exactly wieldy in a melee, but I do think you make a lot of good points about general blacksmith safety precautions.
    $endgroup$
    – Nosajimiki
    2 hours ago













2












2








2





$begingroup$

Treat the wand like it came from a blacksmith's forge



Gloved blacksmith



While some blacksmiths don't use gloves at the forge, others do. And they generally wear them for very short exposures to heat (a wand going whamo should count).



Leather gloves won't be enough to protect against very high heat (fireball spells) but it will protect you from radiant heat near the wand. You also want a leather apron, some kind of safety glasses, and boots.




About the Leather gloves - Experienced blacksmiths will sometimes
recommend no gloves. I strongly recommend a glove on your not-hammer
hand. At least just to start. This will prevent burns. Later, as you
get more experienced with blacksmithing you can decide what you like.



About the boots - This is important because you are going to be
dropping hot pieces of steel and you want a solid pair of boots to
protect your feet from burns. If you plan on handling heavy pieces of
iron and steel then I would go with steel-toed shoes.




So lots of safety gear to start then you can reduce it when you get used to spellcasting.



This site also recommends wolf jaw tongs (not plain jaw/flat). I'd recommend some made special to firmly hold your wand near the base. If blacksmiths can use these to hold on to heavy metal glowing like a miniature sun, and then be able to manipulate the metal on the anvil, you can use them to hold your freaking wand still while you cast.



Remember, the apron and boots and eye coverings (if possible) are for when you are an idiot and drop the hot wand as soon as something blasts out of it.



Tongs



Okay, so this covers you for basic fireballs. Now, what about the Sheeeaaaaah-Moogatee-Hah! spell?



Okay, this requires some setup. I recommend the following:



  • A large barrel made of thick oak plated with metal with a 3-4 inch hole drilled near the top.

  • Fill with water such that it will later reach up to the bottom of the hole.

  • Get in the barrel with something protective on your head.

  • Put on a leather glove.

  • Stick your blacksmithing tongs out the hole and grab the wand.

  • Make sure that under the wand is a bucket of water or a pile of sand.

  • Cast the spell.

  • Drop the wand.

Alternatively...fire that sucker from a stone castle window or rampart, the kind archers shoot out of. Have a trusted running retrieve the wand after it drops in the water/sand and is cool enough to pick up with tongs or a gloved hand.



No, it's not melee-worthy like your fireball and other low to high level spells are. But if you want the nuclear option, you need to build a launch point. This completely violates your requirements, but seriously, do soldiers with fire launchers run around with them in combat? No, they use a sturdy base, sometimes one that is somewhat portable. If you fight with a cannon can you carry it in your pocket then pull it out and boom? Nope.



Remember, your question was protection for a fireball spell. And you state: "casting the fireball spell generates enough heat to cause 3rd degree burns on the unprotected hand and wrist." My solution easily protects against that.






share|improve this answer











$endgroup$



Treat the wand like it came from a blacksmith's forge



Gloved blacksmith



While some blacksmiths don't use gloves at the forge, others do. And they generally wear them for very short exposures to heat (a wand going whamo should count).



Leather gloves won't be enough to protect against very high heat (fireball spells) but it will protect you from radiant heat near the wand. You also want a leather apron, some kind of safety glasses, and boots.




About the Leather gloves - Experienced blacksmiths will sometimes
recommend no gloves. I strongly recommend a glove on your not-hammer
hand. At least just to start. This will prevent burns. Later, as you
get more experienced with blacksmithing you can decide what you like.



About the boots - This is important because you are going to be
dropping hot pieces of steel and you want a solid pair of boots to
protect your feet from burns. If you plan on handling heavy pieces of
iron and steel then I would go with steel-toed shoes.




So lots of safety gear to start then you can reduce it when you get used to spellcasting.



This site also recommends wolf jaw tongs (not plain jaw/flat). I'd recommend some made special to firmly hold your wand near the base. If blacksmiths can use these to hold on to heavy metal glowing like a miniature sun, and then be able to manipulate the metal on the anvil, you can use them to hold your freaking wand still while you cast.



Remember, the apron and boots and eye coverings (if possible) are for when you are an idiot and drop the hot wand as soon as something blasts out of it.



Tongs



Okay, so this covers you for basic fireballs. Now, what about the Sheeeaaaaah-Moogatee-Hah! spell?



Okay, this requires some setup. I recommend the following:



  • A large barrel made of thick oak plated with metal with a 3-4 inch hole drilled near the top.

  • Fill with water such that it will later reach up to the bottom of the hole.

  • Get in the barrel with something protective on your head.

  • Put on a leather glove.

  • Stick your blacksmithing tongs out the hole and grab the wand.

  • Make sure that under the wand is a bucket of water or a pile of sand.

  • Cast the spell.

  • Drop the wand.

Alternatively...fire that sucker from a stone castle window or rampart, the kind archers shoot out of. Have a trusted running retrieve the wand after it drops in the water/sand and is cool enough to pick up with tongs or a gloved hand.



No, it's not melee-worthy like your fireball and other low to high level spells are. But if you want the nuclear option, you need to build a launch point. This completely violates your requirements, but seriously, do soldiers with fire launchers run around with them in combat? No, they use a sturdy base, sometimes one that is somewhat portable. If you fight with a cannon can you carry it in your pocket then pull it out and boom? Nope.



Remember, your question was protection for a fireball spell. And you state: "casting the fireball spell generates enough heat to cause 3rd degree burns on the unprotected hand and wrist." My solution easily protects against that.







share|improve this answer














share|improve this answer



share|improve this answer








edited 3 hours ago









Brythan

21.8k84388




21.8k84388










answered 5 hours ago









CynCyn

13.5k22863




13.5k22863











  • $begingroup$
    Again, partially correct only.
    $endgroup$
    – Hoyle's ghost
    4 hours ago










  • $begingroup$
    @Hoyle'sghost Well write an answer already!
    $endgroup$
    – Cyn
    4 hours ago










  • $begingroup$
    I yet may, though I much prefer to watch others set themselves up for a fall. :)
    $endgroup$
    – Hoyle's ghost
    4 hours ago










  • $begingroup$
    @Hoyle'sghost Ha ha I can tell.
    $endgroup$
    – Cyn
    4 hours ago






  • 1




    $begingroup$
    I think he means because tongs are not exactly wieldy in a melee, but I do think you make a lot of good points about general blacksmith safety precautions.
    $endgroup$
    – Nosajimiki
    2 hours ago
















  • $begingroup$
    Again, partially correct only.
    $endgroup$
    – Hoyle's ghost
    4 hours ago










  • $begingroup$
    @Hoyle'sghost Well write an answer already!
    $endgroup$
    – Cyn
    4 hours ago










  • $begingroup$
    I yet may, though I much prefer to watch others set themselves up for a fall. :)
    $endgroup$
    – Hoyle's ghost
    4 hours ago










  • $begingroup$
    @Hoyle'sghost Ha ha I can tell.
    $endgroup$
    – Cyn
    4 hours ago






  • 1




    $begingroup$
    I think he means because tongs are not exactly wieldy in a melee, but I do think you make a lot of good points about general blacksmith safety precautions.
    $endgroup$
    – Nosajimiki
    2 hours ago















$begingroup$
Again, partially correct only.
$endgroup$
– Hoyle's ghost
4 hours ago




$begingroup$
Again, partially correct only.
$endgroup$
– Hoyle's ghost
4 hours ago












$begingroup$
@Hoyle'sghost Well write an answer already!
$endgroup$
– Cyn
4 hours ago




$begingroup$
@Hoyle'sghost Well write an answer already!
$endgroup$
– Cyn
4 hours ago












$begingroup$
I yet may, though I much prefer to watch others set themselves up for a fall. :)
$endgroup$
– Hoyle's ghost
4 hours ago




$begingroup$
I yet may, though I much prefer to watch others set themselves up for a fall. :)
$endgroup$
– Hoyle's ghost
4 hours ago












$begingroup$
@Hoyle'sghost Ha ha I can tell.
$endgroup$
– Cyn
4 hours ago




$begingroup$
@Hoyle'sghost Ha ha I can tell.
$endgroup$
– Cyn
4 hours ago




1




1




$begingroup$
I think he means because tongs are not exactly wieldy in a melee, but I do think you make a lot of good points about general blacksmith safety precautions.
$endgroup$
– Nosajimiki
2 hours ago




$begingroup$
I think he means because tongs are not exactly wieldy in a melee, but I do think you make a lot of good points about general blacksmith safety precautions.
$endgroup$
– Nosajimiki
2 hours ago











2












$begingroup$

Do what every successful business has ever done with hard to crack problems: outsource it.



In order to outsource the problem of hot wands, you will need:



  • feathers

  • a bow

See a picture below of some witches invoking some explosive fireballs (you know, the kinda that goes boom when it hits something). Notice that they are grasping the wand firmly with the right hand, while grasping the bow with the left hand.



Volley!



Once the wand flies, the problem has been successfully outsourced to whomever you were aiming your fireball at. Any heating concerns are now SEP's (Someone Else's Problem).



And before you tell me that goes against the melee usage rule: you can use an arrow on a melee fight if you really want.






share|improve this answer











$endgroup$








  • 1




    $begingroup$
    Not sure how this would be any better than just dropping it quickly. But a slight variation to this: fire the arrow, and when then wounded person grabs the shaft in reaction to being shot, call out the enchantment blowing up him, and everyone around him. IE: make your enemy grasp the wand
    $endgroup$
    – Nosajimiki
    2 hours ago










  • $begingroup$
    "Not sure how this would be any better than just dropping it quickly." Unless you are dropping the wand from a very high place, this method is different because it also adds maiming to arson.
    $endgroup$
    – Renan
    2 hours ago











  • $begingroup$
    This is an awesome answer, but I find myself wondering about the conductive properties of the wand itself. Ideally, you want your wand to be made of a material that doesn't disperse the heat to the outer surface of the wand for at least a quarter of a second after the spell is cast. Oh, and you've just invented 'Wandcraft' as a surname right alongside 'Fletcher' in the 13th century by turning the manufacture of wands into an industry. Other than that though let the destruction commence!
    $endgroup$
    – Tim B II
    1 hour ago






  • 1




    $begingroup$
    @TimBII You're right. Maybe there is some resin or varnish that could be used around the shaft to keep the heat in; This is not my forte, so I would outsource (and no jokes involved this time) this problem to the local alchemist.
    $endgroup$
    – Renan
    1 hour ago















2












$begingroup$

Do what every successful business has ever done with hard to crack problems: outsource it.



In order to outsource the problem of hot wands, you will need:



  • feathers

  • a bow

See a picture below of some witches invoking some explosive fireballs (you know, the kinda that goes boom when it hits something). Notice that they are grasping the wand firmly with the right hand, while grasping the bow with the left hand.



Volley!



Once the wand flies, the problem has been successfully outsourced to whomever you were aiming your fireball at. Any heating concerns are now SEP's (Someone Else's Problem).



And before you tell me that goes against the melee usage rule: you can use an arrow on a melee fight if you really want.






share|improve this answer











$endgroup$








  • 1




    $begingroup$
    Not sure how this would be any better than just dropping it quickly. But a slight variation to this: fire the arrow, and when then wounded person grabs the shaft in reaction to being shot, call out the enchantment blowing up him, and everyone around him. IE: make your enemy grasp the wand
    $endgroup$
    – Nosajimiki
    2 hours ago










  • $begingroup$
    "Not sure how this would be any better than just dropping it quickly." Unless you are dropping the wand from a very high place, this method is different because it also adds maiming to arson.
    $endgroup$
    – Renan
    2 hours ago











  • $begingroup$
    This is an awesome answer, but I find myself wondering about the conductive properties of the wand itself. Ideally, you want your wand to be made of a material that doesn't disperse the heat to the outer surface of the wand for at least a quarter of a second after the spell is cast. Oh, and you've just invented 'Wandcraft' as a surname right alongside 'Fletcher' in the 13th century by turning the manufacture of wands into an industry. Other than that though let the destruction commence!
    $endgroup$
    – Tim B II
    1 hour ago






  • 1




    $begingroup$
    @TimBII You're right. Maybe there is some resin or varnish that could be used around the shaft to keep the heat in; This is not my forte, so I would outsource (and no jokes involved this time) this problem to the local alchemist.
    $endgroup$
    – Renan
    1 hour ago













2












2








2





$begingroup$

Do what every successful business has ever done with hard to crack problems: outsource it.



In order to outsource the problem of hot wands, you will need:



  • feathers

  • a bow

See a picture below of some witches invoking some explosive fireballs (you know, the kinda that goes boom when it hits something). Notice that they are grasping the wand firmly with the right hand, while grasping the bow with the left hand.



Volley!



Once the wand flies, the problem has been successfully outsourced to whomever you were aiming your fireball at. Any heating concerns are now SEP's (Someone Else's Problem).



And before you tell me that goes against the melee usage rule: you can use an arrow on a melee fight if you really want.






share|improve this answer











$endgroup$



Do what every successful business has ever done with hard to crack problems: outsource it.



In order to outsource the problem of hot wands, you will need:



  • feathers

  • a bow

See a picture below of some witches invoking some explosive fireballs (you know, the kinda that goes boom when it hits something). Notice that they are grasping the wand firmly with the right hand, while grasping the bow with the left hand.



Volley!



Once the wand flies, the problem has been successfully outsourced to whomever you were aiming your fireball at. Any heating concerns are now SEP's (Someone Else's Problem).



And before you tell me that goes against the melee usage rule: you can use an arrow on a melee fight if you really want.







share|improve this answer














share|improve this answer



share|improve this answer








edited 1 hour ago

























answered 2 hours ago









RenanRenan

55.9k15126279




55.9k15126279







  • 1




    $begingroup$
    Not sure how this would be any better than just dropping it quickly. But a slight variation to this: fire the arrow, and when then wounded person grabs the shaft in reaction to being shot, call out the enchantment blowing up him, and everyone around him. IE: make your enemy grasp the wand
    $endgroup$
    – Nosajimiki
    2 hours ago










  • $begingroup$
    "Not sure how this would be any better than just dropping it quickly." Unless you are dropping the wand from a very high place, this method is different because it also adds maiming to arson.
    $endgroup$
    – Renan
    2 hours ago











  • $begingroup$
    This is an awesome answer, but I find myself wondering about the conductive properties of the wand itself. Ideally, you want your wand to be made of a material that doesn't disperse the heat to the outer surface of the wand for at least a quarter of a second after the spell is cast. Oh, and you've just invented 'Wandcraft' as a surname right alongside 'Fletcher' in the 13th century by turning the manufacture of wands into an industry. Other than that though let the destruction commence!
    $endgroup$
    – Tim B II
    1 hour ago






  • 1




    $begingroup$
    @TimBII You're right. Maybe there is some resin or varnish that could be used around the shaft to keep the heat in; This is not my forte, so I would outsource (and no jokes involved this time) this problem to the local alchemist.
    $endgroup$
    – Renan
    1 hour ago












  • 1




    $begingroup$
    Not sure how this would be any better than just dropping it quickly. But a slight variation to this: fire the arrow, and when then wounded person grabs the shaft in reaction to being shot, call out the enchantment blowing up him, and everyone around him. IE: make your enemy grasp the wand
    $endgroup$
    – Nosajimiki
    2 hours ago










  • $begingroup$
    "Not sure how this would be any better than just dropping it quickly." Unless you are dropping the wand from a very high place, this method is different because it also adds maiming to arson.
    $endgroup$
    – Renan
    2 hours ago











  • $begingroup$
    This is an awesome answer, but I find myself wondering about the conductive properties of the wand itself. Ideally, you want your wand to be made of a material that doesn't disperse the heat to the outer surface of the wand for at least a quarter of a second after the spell is cast. Oh, and you've just invented 'Wandcraft' as a surname right alongside 'Fletcher' in the 13th century by turning the manufacture of wands into an industry. Other than that though let the destruction commence!
    $endgroup$
    – Tim B II
    1 hour ago






  • 1




    $begingroup$
    @TimBII You're right. Maybe there is some resin or varnish that could be used around the shaft to keep the heat in; This is not my forte, so I would outsource (and no jokes involved this time) this problem to the local alchemist.
    $endgroup$
    – Renan
    1 hour ago







1




1




$begingroup$
Not sure how this would be any better than just dropping it quickly. But a slight variation to this: fire the arrow, and when then wounded person grabs the shaft in reaction to being shot, call out the enchantment blowing up him, and everyone around him. IE: make your enemy grasp the wand
$endgroup$
– Nosajimiki
2 hours ago




$begingroup$
Not sure how this would be any better than just dropping it quickly. But a slight variation to this: fire the arrow, and when then wounded person grabs the shaft in reaction to being shot, call out the enchantment blowing up him, and everyone around him. IE: make your enemy grasp the wand
$endgroup$
– Nosajimiki
2 hours ago












$begingroup$
"Not sure how this would be any better than just dropping it quickly." Unless you are dropping the wand from a very high place, this method is different because it also adds maiming to arson.
$endgroup$
– Renan
2 hours ago





$begingroup$
"Not sure how this would be any better than just dropping it quickly." Unless you are dropping the wand from a very high place, this method is different because it also adds maiming to arson.
$endgroup$
– Renan
2 hours ago













$begingroup$
This is an awesome answer, but I find myself wondering about the conductive properties of the wand itself. Ideally, you want your wand to be made of a material that doesn't disperse the heat to the outer surface of the wand for at least a quarter of a second after the spell is cast. Oh, and you've just invented 'Wandcraft' as a surname right alongside 'Fletcher' in the 13th century by turning the manufacture of wands into an industry. Other than that though let the destruction commence!
$endgroup$
– Tim B II
1 hour ago




$begingroup$
This is an awesome answer, but I find myself wondering about the conductive properties of the wand itself. Ideally, you want your wand to be made of a material that doesn't disperse the heat to the outer surface of the wand for at least a quarter of a second after the spell is cast. Oh, and you've just invented 'Wandcraft' as a surname right alongside 'Fletcher' in the 13th century by turning the manufacture of wands into an industry. Other than that though let the destruction commence!
$endgroup$
– Tim B II
1 hour ago




1




1




$begingroup$
@TimBII You're right. Maybe there is some resin or varnish that could be used around the shaft to keep the heat in; This is not my forte, so I would outsource (and no jokes involved this time) this problem to the local alchemist.
$endgroup$
– Renan
1 hour ago




$begingroup$
@TimBII You're right. Maybe there is some resin or varnish that could be used around the shaft to keep the heat in; This is not my forte, so I would outsource (and no jokes involved this time) this problem to the local alchemist.
$endgroup$
– Renan
1 hour ago











1












$begingroup$

Freeze it



Before using the wand, freeze it. In a combat situation, this will involve frequent wand switches. Take a tub of ice with you. Store all your wands in the ice, so they are nice and cold. Put on a good pair of gloves. Reach into the tub and grab a wand. Aim and shout the appropriate spell. Immediately drop the wand back into the ice. If your hand is overheated, stick it in the ice. Repeat with the next wand when your hand starts getting cold.



In the thirteenth century your source for ice would be the top of a mountain. Take a hay covered wagon up the mountain to where it gets cold. Make a hole in the center of the hay. Put ice in the hole. Lots of ice. Cover it up with hay from the sides. By the time you get to the bottom of the mountain, you'll have less ice.



Stuff the wands in the ice at any time, even while still at the top of the mountain. You want them to get as cold as possible so that you can maximize the safe power of the spell.






share|improve this answer









$endgroup$












  • $begingroup$
    This appears to be a bit of a problem for Question Limitations, bullet #3, the solution must be practical in a melee situation. But it's a clever idea!
    $endgroup$
    – JBH
    2 hours ago










  • $begingroup$
    Didn't the narrator just "happen to find" a single wand?
    $endgroup$
    – Cyn
    1 hour ago















1












$begingroup$

Freeze it



Before using the wand, freeze it. In a combat situation, this will involve frequent wand switches. Take a tub of ice with you. Store all your wands in the ice, so they are nice and cold. Put on a good pair of gloves. Reach into the tub and grab a wand. Aim and shout the appropriate spell. Immediately drop the wand back into the ice. If your hand is overheated, stick it in the ice. Repeat with the next wand when your hand starts getting cold.



In the thirteenth century your source for ice would be the top of a mountain. Take a hay covered wagon up the mountain to where it gets cold. Make a hole in the center of the hay. Put ice in the hole. Lots of ice. Cover it up with hay from the sides. By the time you get to the bottom of the mountain, you'll have less ice.



Stuff the wands in the ice at any time, even while still at the top of the mountain. You want them to get as cold as possible so that you can maximize the safe power of the spell.






share|improve this answer









$endgroup$












  • $begingroup$
    This appears to be a bit of a problem for Question Limitations, bullet #3, the solution must be practical in a melee situation. But it's a clever idea!
    $endgroup$
    – JBH
    2 hours ago










  • $begingroup$
    Didn't the narrator just "happen to find" a single wand?
    $endgroup$
    – Cyn
    1 hour ago













1












1








1





$begingroup$

Freeze it



Before using the wand, freeze it. In a combat situation, this will involve frequent wand switches. Take a tub of ice with you. Store all your wands in the ice, so they are nice and cold. Put on a good pair of gloves. Reach into the tub and grab a wand. Aim and shout the appropriate spell. Immediately drop the wand back into the ice. If your hand is overheated, stick it in the ice. Repeat with the next wand when your hand starts getting cold.



In the thirteenth century your source for ice would be the top of a mountain. Take a hay covered wagon up the mountain to where it gets cold. Make a hole in the center of the hay. Put ice in the hole. Lots of ice. Cover it up with hay from the sides. By the time you get to the bottom of the mountain, you'll have less ice.



Stuff the wands in the ice at any time, even while still at the top of the mountain. You want them to get as cold as possible so that you can maximize the safe power of the spell.






share|improve this answer









$endgroup$



Freeze it



Before using the wand, freeze it. In a combat situation, this will involve frequent wand switches. Take a tub of ice with you. Store all your wands in the ice, so they are nice and cold. Put on a good pair of gloves. Reach into the tub and grab a wand. Aim and shout the appropriate spell. Immediately drop the wand back into the ice. If your hand is overheated, stick it in the ice. Repeat with the next wand when your hand starts getting cold.



In the thirteenth century your source for ice would be the top of a mountain. Take a hay covered wagon up the mountain to where it gets cold. Make a hole in the center of the hay. Put ice in the hole. Lots of ice. Cover it up with hay from the sides. By the time you get to the bottom of the mountain, you'll have less ice.



Stuff the wands in the ice at any time, even while still at the top of the mountain. You want them to get as cold as possible so that you can maximize the safe power of the spell.







share|improve this answer












share|improve this answer



share|improve this answer










answered 3 hours ago









BrythanBrythan

21.8k84388




21.8k84388











  • $begingroup$
    This appears to be a bit of a problem for Question Limitations, bullet #3, the solution must be practical in a melee situation. But it's a clever idea!
    $endgroup$
    – JBH
    2 hours ago










  • $begingroup$
    Didn't the narrator just "happen to find" a single wand?
    $endgroup$
    – Cyn
    1 hour ago
















  • $begingroup$
    This appears to be a bit of a problem for Question Limitations, bullet #3, the solution must be practical in a melee situation. But it's a clever idea!
    $endgroup$
    – JBH
    2 hours ago










  • $begingroup$
    Didn't the narrator just "happen to find" a single wand?
    $endgroup$
    – Cyn
    1 hour ago















$begingroup$
This appears to be a bit of a problem for Question Limitations, bullet #3, the solution must be practical in a melee situation. But it's a clever idea!
$endgroup$
– JBH
2 hours ago




$begingroup$
This appears to be a bit of a problem for Question Limitations, bullet #3, the solution must be practical in a melee situation. But it's a clever idea!
$endgroup$
– JBH
2 hours ago












$begingroup$
Didn't the narrator just "happen to find" a single wand?
$endgroup$
– Cyn
1 hour ago




$begingroup$
Didn't the narrator just "happen to find" a single wand?
$endgroup$
– Cyn
1 hour ago











1












$begingroup$

I have an answer, but I'm not sure if it breaks the rules or not it goes against the rules on two fronts - drilling holes in the wand (thanks JBH) and violates the 13th century rules (thanks Nosajimiki). But, I'm going to throw it out there anyway as an apocryphal answer.



First of all, you're completely right; this is a classic heat sink problem. But, you're also wrong; a gun barrel heats up because of convection from the gases in the gun barrel; the barrel itself is metal for strength, and also because you actually want the barrel to get hot so that the next bullet doesn't discharge prematurely because of a massive heat buildup in the rifle. This was the whole point of gatling guns back in the day; give each barrel time to cool down before being used to fire another bullet.



In point of fact, the whole problem you have with your wand in terms of thermodynamics is that it isn't like a gun barrel, which would make better sense. What you really want to do is conduct the heat away quickly so that it an dissipate in the atmosphere rather than cooking your hand. This is also why gun stocks and handles are not made of metal - They're designed out of wood, pearl, composites, etc. to shield your hand from the excess heat buildup.



I'm going to assume that your wand is some form of magical wood or other element that traps the heat to insane levels without spontaneously combusting. That is about the only model which could work the way you describe, although it also means that the biggest issue with your wand isn't that it gets hot, it's that it doesn't cool down fast enough. That said, the solution is twofold; firstly you need to reinvent a 'stock', or wand handle, and then you need to borrow an ammunition concept from science fiction.



Put more simply, you need to insulate the hand AND absorb the heat out of the wand fast enough so as not to overwhelm the insulative properties of the stock which you use to grip.



Let's deal with the stock first. If it wasn't for their combustible nature, wood or paper would actually make a good option here. But, in this instance, I'm going to suggest a fibrelass handle with a styrofoam core. This would have several benefits;



. You can turn this into a simple collar or tube which your wand slides into

. The styrofoam will grip the wand quite well, meaning less chance of it slipping out in a fight

. Easily replaceable if the styrofoam starts to wear.



Secondly, we need to create something like a thermal clip for your wand. Basically, the idea of a thermal clip in scifi games is that we have all moved to energy weapons, but those weapons can't disperse their heat so they capture it in a disposable thermal clip, which basically stores the heat and can be replaced quickly in battle. In gaming the idea is to keep the idea of ammunition. In your scenario, it's actually a pretty good analogue for the problem you face.



I'm going to suggest that you drill a small hole down the centre line of your wand (hence the idea that it could be outside the rules) and put a small rod of something like Tungsten in it, with a screw thread on the back end. Why tungsten? Because it has the highest melting point but still pretty good thermal conduction properties. Then what you need is a supply of Liquid N2 bulbs that screw or clip on at the base of the wand.



The idea would be that the tungsten conducts the heat to the bulb, which absorbs the heat and converts the nitrogen back to gas. Once it has done that to all the nitrogen in the bulb, you discard and replace.



Now, I know this isn't a perfect solution; how do you keep your N2 bulbs cold in the field? How many shots can a nitrogen bulb acting as a thermal clip absorb? Seriously, you want me to insert what in my wand? How is that going to affect the magical properties?



I suggest that you get one of your friends who wants to be a 'level 6' magician to assist you with some field trials (preferably one you don't like very much) but ultimately, the engineering of this is quite simple - you need to dissipate heat quickly while insulating your hand from it until the heat has dissipated.



For that reason, gun barrels getting hot is actually a good thing; it's touching them that's bad. Your wand needs a similar dynamic. Putting a styrofoam / fibreglass grip around it will hopefully protect your hand while the 'thermal clip' sucks as much of the excess heat out as quickly as it can, reducing the load on the insulated grip.



In any case, be very careful during your testing and don't let anyone do this who will be indispensable in your future magical career.






share|improve this answer











$endgroup$












  • $begingroup$
    Partially correct only, I'm afraid.
    $endgroup$
    – Hoyle's ghost
    4 hours ago










  • $begingroup$
    @Hoyle'sghost So, which part do you disagree with?
    $endgroup$
    – Tim B II
    4 hours ago










  • $begingroup$
    Several of these materials such as styrofoam, fiberglass, tungsten, and liquid nitrogen violate the 13th century requirement.
    $endgroup$
    – Nosajimiki
    2 hours ago











  • $begingroup$
    You're right, drilling a hole in the wand violates World Rules, bullet #4. However, it does make for a fascinating Steampunk idea.
    $endgroup$
    – JBH
    2 hours ago










  • $begingroup$
    @Nosajimiki - Ha! You're right, I actually went looking for tech level requirements (they're notorious in our questions) and didn't see them in the rules. I should've looked in the actual question... I'll edit up top to reflect this is an apocryphal answer
    $endgroup$
    – Tim B II
    2 hours ago















1












$begingroup$

I have an answer, but I'm not sure if it breaks the rules or not it goes against the rules on two fronts - drilling holes in the wand (thanks JBH) and violates the 13th century rules (thanks Nosajimiki). But, I'm going to throw it out there anyway as an apocryphal answer.



First of all, you're completely right; this is a classic heat sink problem. But, you're also wrong; a gun barrel heats up because of convection from the gases in the gun barrel; the barrel itself is metal for strength, and also because you actually want the barrel to get hot so that the next bullet doesn't discharge prematurely because of a massive heat buildup in the rifle. This was the whole point of gatling guns back in the day; give each barrel time to cool down before being used to fire another bullet.



In point of fact, the whole problem you have with your wand in terms of thermodynamics is that it isn't like a gun barrel, which would make better sense. What you really want to do is conduct the heat away quickly so that it an dissipate in the atmosphere rather than cooking your hand. This is also why gun stocks and handles are not made of metal - They're designed out of wood, pearl, composites, etc. to shield your hand from the excess heat buildup.



I'm going to assume that your wand is some form of magical wood or other element that traps the heat to insane levels without spontaneously combusting. That is about the only model which could work the way you describe, although it also means that the biggest issue with your wand isn't that it gets hot, it's that it doesn't cool down fast enough. That said, the solution is twofold; firstly you need to reinvent a 'stock', or wand handle, and then you need to borrow an ammunition concept from science fiction.



Put more simply, you need to insulate the hand AND absorb the heat out of the wand fast enough so as not to overwhelm the insulative properties of the stock which you use to grip.



Let's deal with the stock first. If it wasn't for their combustible nature, wood or paper would actually make a good option here. But, in this instance, I'm going to suggest a fibrelass handle with a styrofoam core. This would have several benefits;



. You can turn this into a simple collar or tube which your wand slides into

. The styrofoam will grip the wand quite well, meaning less chance of it slipping out in a fight

. Easily replaceable if the styrofoam starts to wear.



Secondly, we need to create something like a thermal clip for your wand. Basically, the idea of a thermal clip in scifi games is that we have all moved to energy weapons, but those weapons can't disperse their heat so they capture it in a disposable thermal clip, which basically stores the heat and can be replaced quickly in battle. In gaming the idea is to keep the idea of ammunition. In your scenario, it's actually a pretty good analogue for the problem you face.



I'm going to suggest that you drill a small hole down the centre line of your wand (hence the idea that it could be outside the rules) and put a small rod of something like Tungsten in it, with a screw thread on the back end. Why tungsten? Because it has the highest melting point but still pretty good thermal conduction properties. Then what you need is a supply of Liquid N2 bulbs that screw or clip on at the base of the wand.



The idea would be that the tungsten conducts the heat to the bulb, which absorbs the heat and converts the nitrogen back to gas. Once it has done that to all the nitrogen in the bulb, you discard and replace.



Now, I know this isn't a perfect solution; how do you keep your N2 bulbs cold in the field? How many shots can a nitrogen bulb acting as a thermal clip absorb? Seriously, you want me to insert what in my wand? How is that going to affect the magical properties?



I suggest that you get one of your friends who wants to be a 'level 6' magician to assist you with some field trials (preferably one you don't like very much) but ultimately, the engineering of this is quite simple - you need to dissipate heat quickly while insulating your hand from it until the heat has dissipated.



For that reason, gun barrels getting hot is actually a good thing; it's touching them that's bad. Your wand needs a similar dynamic. Putting a styrofoam / fibreglass grip around it will hopefully protect your hand while the 'thermal clip' sucks as much of the excess heat out as quickly as it can, reducing the load on the insulated grip.



In any case, be very careful during your testing and don't let anyone do this who will be indispensable in your future magical career.






share|improve this answer











$endgroup$












  • $begingroup$
    Partially correct only, I'm afraid.
    $endgroup$
    – Hoyle's ghost
    4 hours ago










  • $begingroup$
    @Hoyle'sghost So, which part do you disagree with?
    $endgroup$
    – Tim B II
    4 hours ago










  • $begingroup$
    Several of these materials such as styrofoam, fiberglass, tungsten, and liquid nitrogen violate the 13th century requirement.
    $endgroup$
    – Nosajimiki
    2 hours ago











  • $begingroup$
    You're right, drilling a hole in the wand violates World Rules, bullet #4. However, it does make for a fascinating Steampunk idea.
    $endgroup$
    – JBH
    2 hours ago










  • $begingroup$
    @Nosajimiki - Ha! You're right, I actually went looking for tech level requirements (they're notorious in our questions) and didn't see them in the rules. I should've looked in the actual question... I'll edit up top to reflect this is an apocryphal answer
    $endgroup$
    – Tim B II
    2 hours ago













1












1








1





$begingroup$

I have an answer, but I'm not sure if it breaks the rules or not it goes against the rules on two fronts - drilling holes in the wand (thanks JBH) and violates the 13th century rules (thanks Nosajimiki). But, I'm going to throw it out there anyway as an apocryphal answer.



First of all, you're completely right; this is a classic heat sink problem. But, you're also wrong; a gun barrel heats up because of convection from the gases in the gun barrel; the barrel itself is metal for strength, and also because you actually want the barrel to get hot so that the next bullet doesn't discharge prematurely because of a massive heat buildup in the rifle. This was the whole point of gatling guns back in the day; give each barrel time to cool down before being used to fire another bullet.



In point of fact, the whole problem you have with your wand in terms of thermodynamics is that it isn't like a gun barrel, which would make better sense. What you really want to do is conduct the heat away quickly so that it an dissipate in the atmosphere rather than cooking your hand. This is also why gun stocks and handles are not made of metal - They're designed out of wood, pearl, composites, etc. to shield your hand from the excess heat buildup.



I'm going to assume that your wand is some form of magical wood or other element that traps the heat to insane levels without spontaneously combusting. That is about the only model which could work the way you describe, although it also means that the biggest issue with your wand isn't that it gets hot, it's that it doesn't cool down fast enough. That said, the solution is twofold; firstly you need to reinvent a 'stock', or wand handle, and then you need to borrow an ammunition concept from science fiction.



Put more simply, you need to insulate the hand AND absorb the heat out of the wand fast enough so as not to overwhelm the insulative properties of the stock which you use to grip.



Let's deal with the stock first. If it wasn't for their combustible nature, wood or paper would actually make a good option here. But, in this instance, I'm going to suggest a fibrelass handle with a styrofoam core. This would have several benefits;



. You can turn this into a simple collar or tube which your wand slides into

. The styrofoam will grip the wand quite well, meaning less chance of it slipping out in a fight

. Easily replaceable if the styrofoam starts to wear.



Secondly, we need to create something like a thermal clip for your wand. Basically, the idea of a thermal clip in scifi games is that we have all moved to energy weapons, but those weapons can't disperse their heat so they capture it in a disposable thermal clip, which basically stores the heat and can be replaced quickly in battle. In gaming the idea is to keep the idea of ammunition. In your scenario, it's actually a pretty good analogue for the problem you face.



I'm going to suggest that you drill a small hole down the centre line of your wand (hence the idea that it could be outside the rules) and put a small rod of something like Tungsten in it, with a screw thread on the back end. Why tungsten? Because it has the highest melting point but still pretty good thermal conduction properties. Then what you need is a supply of Liquid N2 bulbs that screw or clip on at the base of the wand.



The idea would be that the tungsten conducts the heat to the bulb, which absorbs the heat and converts the nitrogen back to gas. Once it has done that to all the nitrogen in the bulb, you discard and replace.



Now, I know this isn't a perfect solution; how do you keep your N2 bulbs cold in the field? How many shots can a nitrogen bulb acting as a thermal clip absorb? Seriously, you want me to insert what in my wand? How is that going to affect the magical properties?



I suggest that you get one of your friends who wants to be a 'level 6' magician to assist you with some field trials (preferably one you don't like very much) but ultimately, the engineering of this is quite simple - you need to dissipate heat quickly while insulating your hand from it until the heat has dissipated.



For that reason, gun barrels getting hot is actually a good thing; it's touching them that's bad. Your wand needs a similar dynamic. Putting a styrofoam / fibreglass grip around it will hopefully protect your hand while the 'thermal clip' sucks as much of the excess heat out as quickly as it can, reducing the load on the insulated grip.



In any case, be very careful during your testing and don't let anyone do this who will be indispensable in your future magical career.






share|improve this answer











$endgroup$



I have an answer, but I'm not sure if it breaks the rules or not it goes against the rules on two fronts - drilling holes in the wand (thanks JBH) and violates the 13th century rules (thanks Nosajimiki). But, I'm going to throw it out there anyway as an apocryphal answer.



First of all, you're completely right; this is a classic heat sink problem. But, you're also wrong; a gun barrel heats up because of convection from the gases in the gun barrel; the barrel itself is metal for strength, and also because you actually want the barrel to get hot so that the next bullet doesn't discharge prematurely because of a massive heat buildup in the rifle. This was the whole point of gatling guns back in the day; give each barrel time to cool down before being used to fire another bullet.



In point of fact, the whole problem you have with your wand in terms of thermodynamics is that it isn't like a gun barrel, which would make better sense. What you really want to do is conduct the heat away quickly so that it an dissipate in the atmosphere rather than cooking your hand. This is also why gun stocks and handles are not made of metal - They're designed out of wood, pearl, composites, etc. to shield your hand from the excess heat buildup.



I'm going to assume that your wand is some form of magical wood or other element that traps the heat to insane levels without spontaneously combusting. That is about the only model which could work the way you describe, although it also means that the biggest issue with your wand isn't that it gets hot, it's that it doesn't cool down fast enough. That said, the solution is twofold; firstly you need to reinvent a 'stock', or wand handle, and then you need to borrow an ammunition concept from science fiction.



Put more simply, you need to insulate the hand AND absorb the heat out of the wand fast enough so as not to overwhelm the insulative properties of the stock which you use to grip.



Let's deal with the stock first. If it wasn't for their combustible nature, wood or paper would actually make a good option here. But, in this instance, I'm going to suggest a fibrelass handle with a styrofoam core. This would have several benefits;



. You can turn this into a simple collar or tube which your wand slides into

. The styrofoam will grip the wand quite well, meaning less chance of it slipping out in a fight

. Easily replaceable if the styrofoam starts to wear.



Secondly, we need to create something like a thermal clip for your wand. Basically, the idea of a thermal clip in scifi games is that we have all moved to energy weapons, but those weapons can't disperse their heat so they capture it in a disposable thermal clip, which basically stores the heat and can be replaced quickly in battle. In gaming the idea is to keep the idea of ammunition. In your scenario, it's actually a pretty good analogue for the problem you face.



I'm going to suggest that you drill a small hole down the centre line of your wand (hence the idea that it could be outside the rules) and put a small rod of something like Tungsten in it, with a screw thread on the back end. Why tungsten? Because it has the highest melting point but still pretty good thermal conduction properties. Then what you need is a supply of Liquid N2 bulbs that screw or clip on at the base of the wand.



The idea would be that the tungsten conducts the heat to the bulb, which absorbs the heat and converts the nitrogen back to gas. Once it has done that to all the nitrogen in the bulb, you discard and replace.



Now, I know this isn't a perfect solution; how do you keep your N2 bulbs cold in the field? How many shots can a nitrogen bulb acting as a thermal clip absorb? Seriously, you want me to insert what in my wand? How is that going to affect the magical properties?



I suggest that you get one of your friends who wants to be a 'level 6' magician to assist you with some field trials (preferably one you don't like very much) but ultimately, the engineering of this is quite simple - you need to dissipate heat quickly while insulating your hand from it until the heat has dissipated.



For that reason, gun barrels getting hot is actually a good thing; it's touching them that's bad. Your wand needs a similar dynamic. Putting a styrofoam / fibreglass grip around it will hopefully protect your hand while the 'thermal clip' sucks as much of the excess heat out as quickly as it can, reducing the load on the insulated grip.



In any case, be very careful during your testing and don't let anyone do this who will be indispensable in your future magical career.







share|improve this answer














share|improve this answer



share|improve this answer








edited 1 hour ago

























answered 4 hours ago









Tim B IITim B II

34.6k677138




34.6k677138











  • $begingroup$
    Partially correct only, I'm afraid.
    $endgroup$
    – Hoyle's ghost
    4 hours ago










  • $begingroup$
    @Hoyle'sghost So, which part do you disagree with?
    $endgroup$
    – Tim B II
    4 hours ago










  • $begingroup$
    Several of these materials such as styrofoam, fiberglass, tungsten, and liquid nitrogen violate the 13th century requirement.
    $endgroup$
    – Nosajimiki
    2 hours ago











  • $begingroup$
    You're right, drilling a hole in the wand violates World Rules, bullet #4. However, it does make for a fascinating Steampunk idea.
    $endgroup$
    – JBH
    2 hours ago










  • $begingroup$
    @Nosajimiki - Ha! You're right, I actually went looking for tech level requirements (they're notorious in our questions) and didn't see them in the rules. I should've looked in the actual question... I'll edit up top to reflect this is an apocryphal answer
    $endgroup$
    – Tim B II
    2 hours ago
















  • $begingroup$
    Partially correct only, I'm afraid.
    $endgroup$
    – Hoyle's ghost
    4 hours ago










  • $begingroup$
    @Hoyle'sghost So, which part do you disagree with?
    $endgroup$
    – Tim B II
    4 hours ago










  • $begingroup$
    Several of these materials such as styrofoam, fiberglass, tungsten, and liquid nitrogen violate the 13th century requirement.
    $endgroup$
    – Nosajimiki
    2 hours ago











  • $begingroup$
    You're right, drilling a hole in the wand violates World Rules, bullet #4. However, it does make for a fascinating Steampunk idea.
    $endgroup$
    – JBH
    2 hours ago










  • $begingroup$
    @Nosajimiki - Ha! You're right, I actually went looking for tech level requirements (they're notorious in our questions) and didn't see them in the rules. I should've looked in the actual question... I'll edit up top to reflect this is an apocryphal answer
    $endgroup$
    – Tim B II
    2 hours ago















$begingroup$
Partially correct only, I'm afraid.
$endgroup$
– Hoyle's ghost
4 hours ago




$begingroup$
Partially correct only, I'm afraid.
$endgroup$
– Hoyle's ghost
4 hours ago












$begingroup$
@Hoyle'sghost So, which part do you disagree with?
$endgroup$
– Tim B II
4 hours ago




$begingroup$
@Hoyle'sghost So, which part do you disagree with?
$endgroup$
– Tim B II
4 hours ago












$begingroup$
Several of these materials such as styrofoam, fiberglass, tungsten, and liquid nitrogen violate the 13th century requirement.
$endgroup$
– Nosajimiki
2 hours ago





$begingroup$
Several of these materials such as styrofoam, fiberglass, tungsten, and liquid nitrogen violate the 13th century requirement.
$endgroup$
– Nosajimiki
2 hours ago













$begingroup$
You're right, drilling a hole in the wand violates World Rules, bullet #4. However, it does make for a fascinating Steampunk idea.
$endgroup$
– JBH
2 hours ago




$begingroup$
You're right, drilling a hole in the wand violates World Rules, bullet #4. However, it does make for a fascinating Steampunk idea.
$endgroup$
– JBH
2 hours ago












$begingroup$
@Nosajimiki - Ha! You're right, I actually went looking for tech level requirements (they're notorious in our questions) and didn't see them in the rules. I should've looked in the actual question... I'll edit up top to reflect this is an apocryphal answer
$endgroup$
– Tim B II
2 hours ago




$begingroup$
@Nosajimiki - Ha! You're right, I actually went looking for tech level requirements (they're notorious in our questions) and didn't see them in the rules. I should've looked in the actual question... I'll edit up top to reflect this is an apocryphal answer
$endgroup$
– Tim B II
2 hours ago











0












$begingroup$

Sand/Water Cooling



The Heat dissipation doesn't need to be a function of the wand itself. If the user's hands are themselves insulated, they can use traditional cooling techniques. A wand user can carry classical cooling systems as part of their kit. Since I presume this wand doesn't need ammunition, if you equip them similar to a medieval fusilier they can carry sand cartridges/water pouches instead of ammo






share|improve this answer









$endgroup$












  • $begingroup$
    Insulation, yes, partial answer though.
    $endgroup$
    – Hoyle's ghost
    4 hours ago










  • $begingroup$
    This is an interesting idea. How do you use the sand and/or water? Can you add a bit of detail?
    $endgroup$
    – JBH
    2 hours ago















0












$begingroup$

Sand/Water Cooling



The Heat dissipation doesn't need to be a function of the wand itself. If the user's hands are themselves insulated, they can use traditional cooling techniques. A wand user can carry classical cooling systems as part of their kit. Since I presume this wand doesn't need ammunition, if you equip them similar to a medieval fusilier they can carry sand cartridges/water pouches instead of ammo






share|improve this answer









$endgroup$












  • $begingroup$
    Insulation, yes, partial answer though.
    $endgroup$
    – Hoyle's ghost
    4 hours ago










  • $begingroup$
    This is an interesting idea. How do you use the sand and/or water? Can you add a bit of detail?
    $endgroup$
    – JBH
    2 hours ago













0












0








0





$begingroup$

Sand/Water Cooling



The Heat dissipation doesn't need to be a function of the wand itself. If the user's hands are themselves insulated, they can use traditional cooling techniques. A wand user can carry classical cooling systems as part of their kit. Since I presume this wand doesn't need ammunition, if you equip them similar to a medieval fusilier they can carry sand cartridges/water pouches instead of ammo






share|improve this answer









$endgroup$



Sand/Water Cooling



The Heat dissipation doesn't need to be a function of the wand itself. If the user's hands are themselves insulated, they can use traditional cooling techniques. A wand user can carry classical cooling systems as part of their kit. Since I presume this wand doesn't need ammunition, if you equip them similar to a medieval fusilier they can carry sand cartridges/water pouches instead of ammo







share|improve this answer












share|improve this answer



share|improve this answer










answered 6 hours ago









knowadsknowads

1,513421




1,513421











  • $begingroup$
    Insulation, yes, partial answer though.
    $endgroup$
    – Hoyle's ghost
    4 hours ago










  • $begingroup$
    This is an interesting idea. How do you use the sand and/or water? Can you add a bit of detail?
    $endgroup$
    – JBH
    2 hours ago
















  • $begingroup$
    Insulation, yes, partial answer though.
    $endgroup$
    – Hoyle's ghost
    4 hours ago










  • $begingroup$
    This is an interesting idea. How do you use the sand and/or water? Can you add a bit of detail?
    $endgroup$
    – JBH
    2 hours ago















$begingroup$
Insulation, yes, partial answer though.
$endgroup$
– Hoyle's ghost
4 hours ago




$begingroup$
Insulation, yes, partial answer though.
$endgroup$
– Hoyle's ghost
4 hours ago












$begingroup$
This is an interesting idea. How do you use the sand and/or water? Can you add a bit of detail?
$endgroup$
– JBH
2 hours ago




$begingroup$
This is an interesting idea. How do you use the sand and/or water? Can you add a bit of detail?
$endgroup$
– JBH
2 hours ago

















draft saved

draft discarded
















































Thanks for contributing an answer to Worldbuilding Stack Exchange!


  • Please be sure to answer the question. Provide details and share your research!

But avoid


  • Asking for help, clarification, or responding to other answers.

  • Making statements based on opinion; back them up with references or personal experience.

Use MathJax to format equations. MathJax reference.


To learn more, see our tips on writing great answers.




draft saved


draft discarded














StackExchange.ready(
function ()
StackExchange.openid.initPostLogin('.new-post-login', 'https%3a%2f%2fworldbuilding.stackexchange.com%2fquestions%2f147181%2fmanaging-heat-dissipation-in-a-magic-wand%23new-answer', 'question_page');

);

Post as a guest















Required, but never shown





















































Required, but never shown














Required, but never shown












Required, but never shown







Required, but never shown

































Required, but never shown














Required, but never shown












Required, but never shown







Required, but never shown







Popular posts from this blog

ParseJSON using SSJSUsing AMPscript with SSJS ActivitiesHow to resubscribe a user in Marketing cloud using SSJS?Pulling Subscriber Status from Lists using SSJSRetrieving Emails using SSJSProblem in updating DE using SSJSUsing SSJS to send single email in Marketing CloudError adding EmailSendDefinition using SSJS

Кампала Садржај Географија Географија Историја Становништво Привреда Партнерски градови Референце Спољашње везе Мени за навигацију0°11′ СГШ; 32°20′ ИГД / 0.18° СГШ; 32.34° ИГД / 0.18; 32.340°11′ СГШ; 32°20′ ИГД / 0.18° СГШ; 32.34° ИГД / 0.18; 32.34МедијиПодациЗванични веб-сајту

19. јануар Садржај Догађаји Рођења Смрти Празници и дани сећања Види још Референце Мени за навигацијуу